Google Groups no longer supports new Usenet posts or subscriptions. Historical content remains viewable.
Dismiss

Center of Universe?

189 views
Skip to first unread message

Victor Grauer

unread,
Jan 2, 1996, 3:00:00 AM1/2/96
to
This is a question that has bothered me for years --
The farther out we look all around us at the universe, at the
deepest penetration our telescopes allow, the closer we come to the
origin point of the Big Bang. This is easy to grasp, since, at least
according to currently accepted theories due to Hubble, we are looking
not only at distant space but distant time.
Now, according to General Relativity, the universe has no center
in space (the famous balloon analogy). But the expanding universe theory
seems to imply that it does have an *origin* in time and the farther out
we look all around us, the farther back in time we go toward that origin
point.
This seems to lead to some fantastic paradoxes, e.g. the center of
the universe is located at the periphery (which would imply that our
universe is "inside-out"). I know that Hawking has argued that there was
no actual beginning point in time. Perhaps some extrapolation from that
argument would save us here.


Hannu Poropudas

unread,
Jan 4, 1996, 3:00:00 AM1/4/96
to

By the way do you know where in our Universe is the center of
the space (I mean the mass center of all galaxies).?


---


Best Regards,

Hannu Poropudas.

"It's Not What You Know That Matters
... It's Knowing What You Don't."

Emory F. Bunn

unread,
Jan 4, 1996, 3:00:00 AM1/4/96
to
In article <4chedk$i...@geraldo.cc.utexas.edu>,
Miguel Lerma <mle...@arthur.ma.utexas.edu> wrote:
>Hannu Poropudas (hannu.P...@ericsson.fi) wrote:
>
>: By the way do you know where in our Universe is the center of
>: the space (I mean the mass center of all galaxies).?
>
>Probably it is indefined. It is like asking about a center of
>masses of the continents in the surface of the Earth.

That's right. As far as we can tell, the Universe is roughly
homogeneous on large scales. There doesn't seem to be any special
point that you'd want to call "the center." The analogy with the
surface of the Earth is a good one. Let me expand on it just a little
bit, in case anyone missed the point.

The Universe may be spatially finite and yet have no boundary. This
initially may sound impossible, but it's not. The surface of sphere
is a two-dimensional surface that has finite area and yet has no
boundary, and one can imagine analogous three-dimensional "surfaces"
(although such objects are hard to visualize).

If the Universe is of this form, then we can get some intuition about
what's going on by considering the analogy with the spherical
surface. It's crucial to remember that in this analogy the entire
three-dimensional volume of space is being compared to the
two-dimensional surface of the sphere. It may help to imagine a
species of little two-dimensional creatures crawling around on the
surface of the sphere. Since they live in only two dimensions, they
have no idea that their world is actually "curved" into "the third
dimension." As far as they're concerned, that two-dimensional surface
is all there is.

Now, these creatures could do experiments and discover that their
world is shaped like a sphere. They might find this result surprising
at first, but they'd get used to it eventually, and they'd realize
that although their world has finite area, it has no boundary. Nor
does it have a center, at least not one that they can point to. There
is no point on the surface of the sphere that you'd call the center:
all points on the surface look exactly the same. Similarly, if our
three-dimensional Universe is curved like a sphere, it could be finite
and yet have no center.

On the other hand, our Universe might be infinite. We have no way to
tell. If it's infinite, it still might not have a center. It might
just stretch on forever, with each point looking more or less like
every other point.

In fact, if you take our observations of the distribution of stuff in
the Universe, and extrapolate them in the simplest possible way, you
find that these two possibilities (curved like a sphere or extending
forever homogeneously) are the two simplest, most natural hypotheses.
That doesn't necessarily mean that either is right, of course. The
big problem is that we can only see a finite amount of the Universe:
since the Universe is only about 15 billion years old, we can't see
anything further away than about 15 billion light-years. That
distance is known as our "horizon." The Universe seems to be pretty
much homogeneous throughout the volume enclosed by our horizon, so
it's natural to guess that maybe things continue that way outside of
the horizon, but since we can't see out there, that's just a guess.

>You can try to define it as (1/M) SUM r_i m_i, where M is
>the total mass of the universe, m_i is the mass of the i-th
>particle of the universe, r_i is a position vector of that
>particle, and the sum goes throght all the particles in the
>universe (perhaps about 10^80, I am ignoring quantum efects).
>But in a curved space there is no such a thing as "position
>vector". If you substitute it by, say, the lenght of the
>geodesic from some "fix" point taken as origin, the result
>is going to depend on the point chosen.

Absolutely right. But things are even worse than that, because that
number 10^80 is the number of particles in the *observable* Universe
(i.e., within our horizon), not the number in the whole Universe. The
observable Universe is a sphere centered on us, since it's the set of
all points close enough to us for us to see them. So even if you
could get around the problems of spatial curvature, you wouldn't get
reliable results by computing the center of mass of all of the
observable particles. If you did compute such a thing, you'd find
that the center of the Universe was right here (or pretty close to
it), simply because you've artificially restricted your attention to a
sphere centered on us, instead of considering the whole Universe.
An alien in a distant galaxy could perform the same computation
using his own observable Universe, and he'd find that *he*
was at the center instead.

-Ted

Miguel Lerma

unread,
Jan 4, 1996, 3:00:00 AM1/4/96
to
Hannu Poropudas (hannu.P...@ericsson.fi) wrote:

: By the way do you know where in our Universe is the center of
: the space (I mean the mass center of all galaxies).?

Probably it is indefined. It is like asking about a center of
masses of the continents in the surface of the Earth.

You can try to define it as (1/M) SUM r_i m_i, where M is

the total mass of the universe, m_i is the mass of the i-th
particle of the universe, r_i is a position vector of that
particle, and the sum goes throght all the particles in the
universe (perhaps about 10^80, I am ignoring quantum efects).
But in a curved space there is no such a thing as "position
vector". If you substitute it by, say, the lenght of the
geodesic from some "fix" point taken as origin, the result
is going to depend on the point chosen.


Miguel A. Lerma


Andrew Cooke

unread,
Jan 4, 1996, 3:00:00 AM1/4/96
to
In article <4cbi01$r...@titania.pps.pgh.pa.us>,

everything you say up to the last paragraph is more-or-less
correct (in my humble opinion!). the `fantastic paradoxes'
i think you are noticing are:

- spacetime is curved. if the universe is smaller at earlier times
then it must have a smaller volume. so volume is not increasing
as we go further and further away (back in time). one effect of
this is that the angular size of galaxies stops getting smaller
as they get further away(!). the details depend on which model
of the universe you use, but if you want more info check out
`the angular diameter distance relation' in astronomy textbooks.

- even at the big bang, for a flat or open universe, the spatial
extent of the universe is infinite. this gives me a headache if
i think about it for too long, but you might consider it
`paradoxical'....

don't forget that as we approach the moment of the big-bang
the physics becomes less and less well known as the energies get
higher and higher. the `big bang' is only a model and it becomes
more and more uncertain as we approach the initial moment
(incidentally, if anyone out there knows, to what extent are
singularity theorems immune to changing physics?)

andrew

--
work phone/fax: 0131 668 8356, office: 0131 668 8357
institute for astronomy, royal observatory, blackford hill, edinburgh
http://www.roe.ac.uk/ajcwww

Andrew Cooke

unread,
Jan 4, 1996, 3:00:00 AM1/4/96
to
In article <4ch3lc$d...@scotsman.ed.ac.uk>,

Andrew Cooke <A.C...@roe.ac.uk> wrote:
> - spacetime is curved. if the universe is smaller at earlier times
> then it must have a smaller volume. so volume is not increasing

...as fast as you would expect...

> as we go further and further away (back in time). one effect of
> this is that the angular size of galaxies stops getting smaller
> as they get further away(!).

andrew

(just thought i better correct that!)

Edward Green

unread,
Jan 4, 1996, 3:00:00 AM1/4/96
to
'gra...@oberon.pps.pgh.pa.us (Victor Grauer)' wrote:

>This is a question that has bothered me for years --

Perhaps you will let me clear some things up then?

> The farther out we look all around us at the universe, at the
>deepest penetration our telescopes allow, the closer we come to the
>origin point of the Big Bang. This is easy to grasp, since, at least
>according to currently accepted theories due to Hubble, we are looking
>not only at distant space but distant time.

I don't know what you mean by "theories due to Hubble", but "looking
at distant time" just follows from the finite speed of light.

> Now, according to General Relativity, the universe has no center
>in space (the famous balloon analogy).

General Relativity is in the first place a local theory of space and
matter. What you describe may be a feature of a particular *solution* to
the field equations, but so far as I know is not a necessary feature of
all solutions.

>But the expanding universe theory
>seems to imply that it does have an *origin* in time and the farther out
>we look all around us, the farther back in time we go toward that origin
>point.
> This seems to lead to some fantastic paradoxes, e.g. the center of
>the universe is located at the periphery (which would imply that our
>universe is "inside-out").

To return to your "famous balloon analogy", it's apparent that by looking
out in all directions, we are really be looking at the same distant region
of space. We cannot look far enough out to see times right after the Big
Bang, so we cannot see all the way around the balloon to a point, but
merely see the perimeter of some "hidden region" (spooky).

Now, even assuming a universe with this topology, the distant region we
see in all directions is by no means special... it's more like our
antipode on the balloon. We happen to see it an earlier epoch, but it's
not "closer" to the "center". Heck, you just got through telling us that
the universe has no center :-) The Big Bang "happened" "everywhere" at
once, and the residual radiation that is supposed to be its signature is
permeating space.

In a nutshell, your "center is at the periphery" paradox mixes time and
space. Even relativity preserves *some* distinction. You can forget about
the balloon and see the situation clearly in a one-dimensional "rubber
band" universe, with signals of finite speed propagating around the band.
The Big Bang is the extrapolated time when the infinitesimal band first
popped into being. :-)

Suggestions for further handwaving:

Since we can't see 'round and 'round our universe, does this mean the
expansion was at some time faster than light? Or that now distant regions
of space have never communicated since the dawn of time? Strange. Then
again, everything in cosmology is strange. All cosmologies are
inconceivable, and yet one simply *is*.

Disclaimer:

This reply has started in certainty and ended in speculation. An
increasing density of "so far as I know" is to be assumed to the extent
necessary to vacate false assertions. :-)

>...Hawking has argued that there was
>no actual beginning point in time. Perhaps some extrapolation from that
>argument would save us here.

Save us from what?

--

Ed Green
egr...@nyc.pipeline.com

Andrew Cooke

unread,
Jan 5, 1996, 3:00:00 AM1/5/96
to
In article <4cj3u2$m...@pipe10.nyc.pipeline.com>,
Edward Green <egr...@nyc.pipeline.com> wrote:

>'t...@physics2.berkeley.edu (Emory F. Bunn)' wrote:
>
>>The
>>big problem is that we can only see a finite amount of the Universe:
>>since the Universe is only about 15 billion years old, we can't see
>>anything further away than about 15 billion light-years.
>
>Then I misspoke. So we can see features that were formed right after the
>big bang.

just after the `big bang' everything was so hot that photons were
continually scattering off particles (mainly electrons).

as things expanded and cooled this became less important, and
the effective change was actually quite sudden - so we can see
back to a certain point, but then no further because it is
`misty' due to all the scattering.

this `misty' early universe is what people are looking at when
they look at fluctuations in the microwave background. since
the universe was still expanding very rapidly at that time it
is at a very high redshift (about 1000 if i remember correctly).

that's why people were so interested in the fluctuations - they
were the very beginnings of the structures that are now galaxies.
and they are only just big enough (if they hadn't been seen people
would have had big problems because it would have been too smooth
to form the universe we have today).

andrew

Richard A. Schumacher

unread,
Jan 5, 1996, 3:00:00 AM1/5/96
to
In <4cif07$f...@lmfpub.lmf.ericsson.se> Hannu.P...@ericsson.fi (Hannu Poropudas) writes:

>Earth have a center, but it is outside of Earth's surface.


Correct! The center of Earth's surface is not on the surface.
And the same is true of the universe: the center of the universe
is not in the universe. It lies outside the universe, 15 billion
or so years in the past. The center is not "near the center of
Virgo Super Cluster" or anywhere else accessible to us.

Paul J. Kossick

unread,
Jan 5, 1996, 3:00:00 AM1/5/96
to
I think the question might be better stated as: Where would be the center
of the universe in 3-dimensional space? To answer this would require
knowledge of the actual 'shape' of the universe in that space, and I
don't believe anyone actually agrees on that.

--
*-*-*-*-*-*-*-*-*-*-*-*-*-*-*-*-*-*-*-*-*-*-*-*-*-*-*-*-*-*-*-*-*-*-*-*-*-*
Paul J. Kossick Standing on a hill in my mountain of dreams
kos...@crl.com Telling myself it's not as hard, hard, hard as it seems
*-*-*-*-*-*-*-*-*-*-*-*-*-*-*-*-*-*-*-*-*-*-*-*-*-*-*-*-*-*-*-*-*-*-*-*-*-*


Hannu Poropudas

unread,
Jan 5, 1996, 3:00:00 AM1/5/96
to

In article <4chedk$i...@geraldo.cc.utexas.edu>
which is dated 4 Jan 1996 20:47:48 GMT
mle...@arthur.ma.utexas.edu (Miguel Lerma) wrote:

>Hannu Poropudas (hannu.P...@ericsson.fi) wrote:
>
>: By the way do you know where in our Universe is the center of
>: the space (I mean the mass center of all galaxies).?
>
>Probably it is indefined. It is like asking about a center of
>masses of the continents in the surface of the Earth.
>

Earth have a center, but it is outside of Earth's surface.

Perhaps center of the space (center of all galaxies) is
outside of visible Universe, perhaps in the side of
contracting part of it (please take a look of README-articles
in my directory mentioned below).?

Perhaps it could be somewhere near center of Virgo Super Cluster.
Perhaps in invisible center of M87.?

>You can try to define it as (1/M) SUM r_i m_i, where M is
>the total mass of the universe, m_i is the mass of the i-th
>particle of the universe, r_i is a position vector of that
>particle, and the sum goes throght all the particles in the
>universe (perhaps about 10^80, I am ignoring quantum efects).
>But in a curved space there is no such a thing as "position
>vector". If you substitute it by, say, the lenght of the
>geodesic from some "fix" point taken as origin, the result
>is going to depend on the point chosen.
>
>
>Miguel A. Lerma


Sure there is no such a thing as "position vector" in a curved
space. You have to generalize vector concept to be as "directed
geodesic line" in a curved space. You have to define also how
to add, subtract and multiply by scalar these "directed geodesic
lines". For example on two dimensional sphere surface you can
define multiplication and division of these "directed geodesic
lines". One problem remain and that is how to define these
four operations for these objects which starts from different
points on the sphere. Also this "algebra" has two characteristic
features, namely non-associativity and non-distributivity.
Please take a look at my trial in WWW.FUNET.FI (or FTP.FUNET.FI)
and directory pub/doc/misc/HannuPoropudas article is PostScript
file called surface_algebras.Z.

David Jacob

unread,
Jan 5, 1996, 3:00:00 AM1/5/96
to

On 4 Jan 1996, Hannu Poropudas wrote:

>
> By the way do you know where in our Universe is the center of
> the space (I mean the mass center of all galaxies).?
>
>

> ---
>
>
> Best Regards,
>
> Hannu Poropudas.

In the beginning, the Universe issued from a single point, and was
composed of a at least 11 dimensions. Then the universe became unstable,
and all but 3 of the dimensions collapsed, releasing tremendous energies,
and causing the center of the universe to actualy detach and become a
free wandering anomolie. The Center of the Universe has been wandering
around ever since, and has been sought after by the various civilizations
and species of the galaxy since the dawn of life. It is said that
whomsoever possesses the true center of the universe possesses infinite
power to reshape reality and bend others to his will...

Dave

ps--if any hollywood types wish to use the above for the
plot of a movie or something, dont forget my check...


Miguel Lerma

unread,
Jan 5, 1996, 3:00:00 AM1/5/96
to
Hannu Poropudas (Hannu.P...@ericsson.fi) wrote:
[...]
: Earth have a center, but it is outside of Earth's surface.

I guess the closest to this idea is what Richard A. Schumacher says
in his post: the center is 15 million years in the past (Big Bang).
But I think this assumes that only the 3-space is curved, and that
the whole 4-dimensional manifold made up with the whole past, present
and future history of the universe is Euclidean. It is still possible
that the universe is even more complicated than that. I think Hawking
suggested that the whole 4-dimensional manifold is curved, and that
the Big Bang and the Big Cruch are just points like any other
else except for the fact that in a certain system of cordinates
all coordinates except one (i.e.: the three spacial coordinates
but not time) converge there (something similar to what happens
in the surface of the Earth with meridians and paralels). If that
is so, then the "center" of the universe can be conceived only
with help of a fith dimension, so than the universe is a 4-sphere
inmersed in a 5-dimencional Euclidean space.


Miguel A Lerma

Emory F. Bunn

unread,
Jan 5, 1996, 3:00:00 AM1/5/96
to
In article <4cjt0c$4...@crl2.crl.com>, Paul J. Kossick <kos...@crl.com> wrote:
>I think the question might be better stated as: Where would be the center
>of the universe in 3-dimensional space? To answer this would require
>knowledge of the actual 'shape' of the universe in that space, and I
>don't believe anyone actually agrees on that.

I'm not sure I understand what you mean here. Space, as far as we can
tell, looks like a three-dimensional manifold. It fills all of
three-dimensional space, more or less by definition, and so it doesn't
have a "shape in 3-dimensional space." It seems to me that one only
talks about the "shape" of something if the something is embedded in
some larger volume. For example, we might talk about the shape of a
spherical volume that's embedded in a larger three-dimensional space.
But since the Universe doesn't seem to be embedded in a larger
three-dimensional space, I don't know what meaning one would attach to
a phrase like "the shape of the Universe in three-dimensional space."

If you like, you can choose to imagine that the Universe is embedded
in some space of a larger number of dimensions. This is a useful
thing to do when trying to get some intuition about curved spacetime,
although we have no particular reason to believe that space really is
embedded in such a way. In this context, it would make sense to talk
about the shape of the Universe within this larger (four or more
dimensional) space.

-Ted

Edward Green

unread,
Jan 5, 1996, 3:00:00 AM1/5/96
to
't...@physics2.berkeley.edu (Emory F. Bunn)' wrote:

>The
>big problem is that we can only see a finite amount of the Universe:
>since the Universe is only about 15 billion years old, we can't see
>anything further away than about 15 billion light-years.

Then I misspoke. So we can see features that were formed right after the
big bang.

--

Ed Green
egr...@nyc.pipeline.com

Andrew Cooke

unread,
Jan 5, 1996, 3:00:00 AM1/5/96
to
In article <4cjj8k$7...@pipe11.nyc.pipeline.com>,
Edward Green <egr...@nyc.pipeline.com> wrote:

>'a...@reaxp01.roe.ac.uk (Andrew Cooke)' wrote:
>> as things expanded and cooled this became less important, and
>> the effective change was actually quite sudden - so we can see
>> back to a certain point, but then no further because it is
>> `misty' due to all the scattering.
>The I assume this era was relatively brief by current models. A million
>years? A thousand? A nanosecond?

i can't remember. certainly not long compared to the age of
the universe. it would also depend on the details of the model
(density, inflation, etc).

i *should know* (and so should my office mate!) - i'll check it
when i'm next in the library.

>> this `misty' early universe is what people are looking at when
>> they look at fluctuations in the microwave background. since
>> the universe was still expanding very rapidly at that time it
>> is at a very high redshift (about 1000 if i remember correctly).
>

>I though I remembered the explanation that this radiation had "cooled".
>Now I suppose cooling can be distinguished form a straight red-shift by
>spectral signature. This is the "4K" background radition I take it.
>Comment?

yes, it was cooling and expanding and yes, it's the 4(3?)K
background - at a redshift of 1000 it would be at a temperature
of 4 x 10^12 K - a lot hotter!

when i say `redshift of 1000' i mean that if there was something
at that distance then, if we could observe a recognisable
spectrum, it would have a redshift of 1000 (ish). in practice
you are not going to able to do that because there wasn't any
significant line emission (the universe was in almost perfect
thermal equilibrium?). so all i am doing is giving the redshift
from a mathematical model, not an observed measurement. the
most distant things we *observe* (apart from the structure in
the microwave background) have a redshift of about 5.

for a `black body' (which is the spectrum of the background) you
can't measure a redshift - at different redshifts you just have
different temperature black bodies!

hope that makes sense,

MARSHALL DUDLEY

unread,
Jan 5, 1996, 3:00:00 AM1/5/96
to
hannu.P...@ericsson.fi (Hannu Poropudas) writes:

-> By the way do you know where in our Universe is the center of
-> the space (I mean the mass center of all galaxies).?

This is an interesting question. I believe some of the hyperspace equations
show that if the universe is closed then there is no center, or alternative
every point can be viewed as the center. Of course to see this requires
viewing in more than 3 dimensions, but an analogy is to view the earth's 2
dimensions from 3 dimensions, and then it is obvious that no point on the
surface of the earth is the center, and that every point can also be viewed as
the center, since all other points on the surface are symetrically surrounding
it.

Marshall

Edward Green

unread,
Jan 5, 1996, 3:00:00 AM1/5/96
to
'a...@reaxp01.roe.ac.uk (Andrew Cooke)' wrote:

> just after the `big bang' everything was so hot that photons
were
> continually scattering off particles (mainly electrons).
>
> as things expanded and cooled this became less important, and
> the effective change was actually quite sudden - so we can see
> back to a certain point, but then no further because it is
> `misty' due to all the scattering.

The I assume this era was relatively brief by current models. A million
years? A thousand? A nanosecond?

> this `misty' early universe is what people are looking at when
> they look at fluctuations in the microwave background. since
> the universe was still expanding very rapidly at that time it
> is at a very high redshift (about 1000 if i remember correctly).

I though I remembered the explanation that this radiation had "cooled".
Now I suppose cooling can be distinguished form a straight red-shift by
spectral signature. This is the "4K" background radition I take it.
Comment?

> that's why people were so interested in the fluctuations - they
> were the very beginnings of the structures that are now galaxies.
> and they are only just big enough (if they hadn't been seen people
> would have had big problems because it would have been too smooth
> to form the universe we have today).

--

Ed Green
egr...@nyc.pipeline.com

Bruce Scott TOK

unread,
Jan 5, 1996, 3:00:00 AM1/5/96
to
Edward Green (egr...@nyc.pipeline.com) wrote:

: 'a...@reaxp01.roe.ac.uk (Andrew Cooke)' wrote:
:
: > just after the `big bang' everything was so hot that photons
: were
: > continually scattering off particles (mainly electrons).
: >
: > as things expanded and cooled this became less important, and
: > the effective change was actually quite sudden - so we can see
: > back to a certain point, but then no further because it is
: > `misty' due to all the scattering.
:
: The I assume this era was relatively brief by current models. A million
: years? A thousand? A nanosecond?

A few hundred thousand years. Pretty brief.

:
: > this `misty' early universe is what people are looking at when

: > they look at fluctuations in the microwave background. since
: > the universe was still expanding very rapidly at that time it
: > is at a very high redshift (about 1000 if i remember correctly).
:
: I though I remembered the explanation that this radiation had "cooled".
: Now I suppose cooling can be distinguished form a straight red-shift by
: spectral signature. This is the "4K" background radition I take it.
: Comment?

Yes. The temperature at recombination was some 3000 to 4000 K (it helps
to look things up; I almost told you the time at recombination was 3000
years, but that was the temperature I had remembered :-).

The cooling is different from a red shift. A red shift tells you a
relative velocity, while the temperature refers to a _random_ velocity
(in this case, momentum, since we're speaking of photons after
recombination).

BTW... recombination refers to the capturing of the free electrons in
the 'cosmic soup' by protons. Before this time, you had several species
of particle -- which mix is a function of the temperature and density --
all interacting via scattering. This keeps all the constituent
temperatures the same. So right before recombination everything had the
same temperature -- about 3000 K -- and right afterward, the radiation
bath and hydrogen bath still had the same temperature. But afterward
they evolved independently; the photon bath cools as the 'scale factor'
R expands (T is inversely proportional to R).

It is called "re"-combination because the physical process involved is
seen in the lab after you ionise a gas. A bit of historical baggage.

source:

J N Islam, An Introduction to Mathematical Cosmology (Cambridge, 1992)

--
Mach's gut!
Bruce Scott The deadliest bullshit is
Max-Planck-Institut fuer Plasmaphysik odorless and transparent
b...@ipp-garching.mpg.de -- W Gibson

Victor Grauer

unread,
Jan 6, 1996, 3:00:00 AM1/6/96
to
If, at the farthest reaches of space in all directions we see extremely
red shifted photons which are relics of a time very close to the time of
the big bang, then, to me, this clearly means that the center of the
universe is at the periphery.

Bruce Scott TOK

unread,
Jan 6, 1996, 3:00:00 AM1/6/96
to
Victor Grauer (gra...@oberon.pps.pgh.pa.us) wrote:
: If, at the farthest reaches of space in all directions we see extremely
: red shifted photons which are relics of a time very close to the time of
: the big bang, then, to me, this clearly means that the center of the
: universe is at the periphery.

Don't forget you are looking back in time. As someone else said, this
means the center of the universe is its beginning.

[At t=0 in the standard model, the curvature is infinite for all values
of the other three coordinates.]

Emory F. Bunn

unread,
Jan 6, 1996, 3:00:00 AM1/6/96
to
In article <4cm998$j...@titania.pps.pgh.pa.us>,

Victor Grauer <gra...@oberon.pps.pgh.pa.us> wrote:
>If, at the farthest reaches of space in all directions we see extremely
>red shifted photons which are relics of a time very close to the time of
>the big bang, then, to me, this clearly means that the center of the
>universe is at the periphery.

That conclusion doesn't follow. When we look at the farthest reaches
of space, we are looking far into the past, since light from those
points has taken a long time to reach us. In fact, the limit on the
furthest points we can see is set by the age of the Universe: we can't
see objects further than about 15 billion light-years, since light
from more distant objects hasn't had time to reach us.

So when we look at the furthest objects we can see, we're necessarily
going to be seeing them as they were when the Universe was very
young. That's why points near the edge of the observable Universe
look like they're "close to the big bang". They're not *spatially*
any closer to the center than we are. The light we see from those
points did originate from *times* close to the big bang, but that's
just because we happen to be looking at those points from very far
away; it's not anything special about that region of space.

Here's another way to put it. At this very moment there could be a
race of creatures in a galaxy at the edge of our observable Universe.
If they look our way, they will see radiation that left our patch of
space shortly after the moment of the big bang. They might conclude
that our patch of space is "close to the big bang", just as we might
conclude the same thing from our observations of their patch of
space. But we'd both be wrong.

-Ted

Edward Green

unread,
Jan 6, 1996, 3:00:00 AM1/6/96
to
On Jan 05, 1996 17:26:27 in article <Re: Center of Universe?>,

'a...@reaxp01.roe.ac.uk (Andrew Cooke)' wrote:
>
> for a `black body' (which is the spectrum of the background) you
> can't measure a redshift - at different redshifts you just have
> different temperature black bodies!

Yes, I see what you mean. A constant factor in the frequency cannot be
distinguished from an equivalent change in temperature. My thanks to Emory
Bunn also for mentioning this.

'b...@ipp-garching.mpg.de (Bruce Scott TOK )' was kind enough to look up

some numbers and also wrote:

>The cooling is different from a red shift. A red shift tells you a
>relative velocity, while the temperature refers to a _random_ velocity
>(in this case, momentum, since we're speaking of photons after
>recombination).

Conceptually different, no doubt. But practically, if we can't
distinguish red shift from cooling, is that trying to tell us something?
Or this only an incredible coincidence brought on by lack of mathematical
imagination on the part of the universe?

(Not a rhetorical question, but I don't necessarily expect an answer)

Now, to shift gears, I've been trying my hand as a pro-aether crank
recently :-) , and when

't...@physics2.berkeley.edu (Emory F. Bunn)' writes:

>All that matters is that
>the expansion of the Universe stretched the wavelengths of all of
>those photons by a factor of 1100.

why that just gives me verbal fodder!

From a cosmological point of view it's apparent we regard the idea that
space is a *stuff* and matter in energy are embedded in that *stuff* as the
mildest milque-toast; indeed, isn't that the meta-model of GR, which
Einstein is supposed to have regarded as evidence for an aether?

This successful 'reification' of space seems to stand in spendid isolation,
a problem we sometimes describe as requiring the "quantization of
gravity", but may equally well require the "general relativization of the
remaining forces" . (Let me put myself out on a limb; when this happens,
I think virtual particles will be seen as a calculational method only)

Someday I hope to tell you what I think a string is, but not too soon.
I just can't model it mathematically at the moment. :-( :-(

--

Ed Green
egr...@nyc.pipeline.com

Miguel Lerma

unread,
Jan 6, 1996, 3:00:00 AM1/6/96
to
Emory F. Bunn (t...@physics2.berkeley.edu) wrote:
> In article <4ck30f$b...@geraldo.cc.utexas.edu>,

> Miguel Lerma <mle...@arthur.ma.utexas.edu> wrote:
> >I guess the closest to this idea is what Richard A. Schumacher says
> >in his post: the center is 15 million years in the past (Big Bang).

> You mean "billion" rather than "million." Specifically, I should
> point out for the benefit of non-U.S. readers that I mean a
> U.S. billion, 1000000000, not a U.K. billion, which is 1000 times
> bigger. (I think even Nature uses "billion" in the U.S. sense
> now, by the way.)

Right, I meant (US) "billion". Sorry.

> >But I think this assumes that only the 3-space is curved, and that
> >the whole 4-dimensional manifold made up with the whole past, present
> >and future history of the universe is Euclidean.

> I can't speak for Richard Shumacher, but I can tell you that this isn't
> how I interpreted what he wrote. Standard theories of cosmology
> are based on the theory of general relativity. In general relativity,
> spacetime is modeled as a four-dimensional manifold, but definitely
> not a Euclidean one. Spacetime has curvature in all of these models.
[...]

Your remarks are perfectly sound. I was just playing around with
"naive" models of the universe, just to see in what extent they
could provide some meaning to the original question. In particular,
the idea that places the "center" of the universe in the Big Bang
comes from a model in which the universe is like a balloon growing
from an initial point, and the radial coordinate would be the time.
The center would correspond to t=0. Of course, if we want to
deal with "state of the art" models of the universe, we need to
look at relativistic cosmology.

By the way, I have always found intriguing the relation between
the local structure of the universe (as a differenciable manifold)
and its global topology. I think most of the time cosmologists make
implicit assumptions about how they are related. In particular they
estimate the size of the universe from its local curvature. However,
it seems to me that they are different problems. I can imagine, for
instance, manifolds of zero curvature and finite size, e.g. a plane
torus. Also I can conceive, say, infinite 2-manifolds with constant
positive curvature. A sofisticated but interesting example is the
following: let H be the the open upper half complex plane, Q the set of
rational numbers, H-hat = H union Q union {infinity}, j: H-hat -> P^1(C)
the j-invariant (P^1(C) is the Riemann sphere, j appears in the theory
of modular forms), and s: P^1(C) -> S^2 a stereographic projection. S^2
(2-sphere) is assumed to have its usual constant curvature differential
structure. Recall that j is invariant by the modular group, and that H-hat
can be partitioned into infinitely many fundamental domains. The interesting
thing is that the H-hat can be endowed with a contant positive curvature
differential structure via the map z -> s(j(z)), and each fundamental
domain maps bijectively to S^2. It is like having infinitely 2-spheres
glued together in a single 2-manifold.

In short, I do not think that the local structure of the universe
allows us to get conclusions about its global structure without
additional assumptions.


Miguel A. Lerma


Emory F. Bunn

unread,
Jan 6, 1996, 3:00:00 AM1/6/96
to
In article <4cmo6n$2...@geraldo.cc.utexas.edu>,
Miguel Lerma <mle...@arthur.ma.utexas.edu> wrote:

>By the way, I have always found intriguing the relation between
>the local structure of the universe (as a differenciable manifold)
>and its global topology. I think most of the time cosmologists make
>implicit assumptions about how they are related. In particular they
>estimate the size of the universe from its local curvature. However,
>it seems to me that they are different problems. I can imagine, for
>instance, manifolds of zero curvature and finite size, e.g. a plane
>torus.

You're absolutely right. The "standard model" of cosmology
involves a couple of assumptions that people sometimes don't bother
to state explicitly:

1. The density is roughly uniform over very large scales
2. The Universe is simply connected.

There's pretty good evidence for 1, at least over the scales that we
can observe. On the other hand, if the Universe is much larger than
our horizon, it's quite plausible that the density might vary
dramatically over scales of, say, a trillion light-years, and we'd
never know it. So even if someone found an incredibly clever way to
measure the density within our horizon and thereby established that
space was negatively curved around here, that wouldn't prove that the
Universe was truly open, since openness is a global property that
depends on what things look like at arbitrarily large distances.

The second assumption (the one about topology) is the one you were
talking about. Cosmologists usually say that if the Universe has zero
or negative curvature, then it goes on forever. That conclusion
depends on assumption 2. For both flat and negatively curved models,
the only simply connected topology is the one that goes on forever,
but in both cases you can change the topology to get a compact space.
(In the flat case, the simplest way is to make space a 3-torus, as you
say. If there's uniform negative curvature then you have to go to
more complicated topologies.)

There is a relatively small literature of attempts to place
constraints on these alternative topologies. As far as I know, only
the flat toroidal case has been considered, since the open case is
much more complicated. As you'd expect, if the size of the torus is
much larger than our horizon, there's no way you can tell you're in a
torus rather than an infinite space. If the size is comparable to the
horizon or smaller, then there are observational tests you can
perform. If anyone is really interested, I can look up the limits
that have been placed on the torus size in these models. (I think the
length of the torus is constrained to be larger than something like
0.2 to 0.5 horizon sizes.)

>Also I can conceive, say, infinite 2-manifolds with constant
>positive curvature.

That's interesting. I didn't know that such things existed. I'm
afraid I didn't really understand your construction of such a thing on
first reading; I'll try to look at it more carefully later.

I have the impression that you can't make a noncompact 3-manifold with
constant positive curvature. Do you know if that's true or not? (I'm
a couple of miles from my book on Riemannian geometry at the moment;
I'll try to look it up later if I remember.) If that's true, then the
conventional wisdom that locally positive curvature implies a finite
Universe depends only on assumption 1 above, not on assumption 2.

(Note that we cosmologists think we're doing pretty well if our
conclusions depend on only *one* wholly unverifiable assumption! :-)


>In short, I do not think that the local structure of the universe
>allows us to get conclusions about its global structure without
>additional assumptions.

Agreed. For what it's worth, most working cosmologists know
this, although we're frequently to careless to say so.

-Ted

Miguel Lerma

unread,
Jan 7, 1996, 3:00:00 AM1/7/96
to
Emory F. Bunn (t...@physics2.berkeley.edu) wrote:
[...]

> >Also I can conceive, say, infinite 2-manifolds with constant
> >positive curvature.

> That's interesting. I didn't know that such things existed. I'm
> afraid I didn't really understand your construction of such a thing on
> first reading; I'll try to look at it more carefully later.

I mentioned that example because it is related to a problem I have
been studying recently, but there are simpler examples. However
one should be cautious, because that kind of surface is not
completely homogeneous, it contains some exceptional points
(perhaps I abused the language by calling it "manifold"). For instance,
consider the Riemann surface for f(z) = sqrt(z). It can be seen as two
Riemann spheres glued along the negative real axis. If you identify each
of those Riemann spheres with S^2 and look at its differential structure,
you see an object of constant positive curvature with the size of two
spheres of the same curvature. But it contains two branching points:
0 and infinity. Almost everywhere that manifold looks like a sphere,
but at z=0 (and at z=infinity) little circles surounding that point
have length close to 4*pi*r instead of 2*pi*r.

In my example, the surface can be seen as infinitely many
spheres glued in a certain way along the negative real axis
and the interval [0,1728]. At almost every point that manifold
looks like a piece of sphere, but the point 0 is exceptional.
If you are close to z=0 in one of the spheres and go around it
in a small circle, you will go through six diferent spheres
making an arc of 180 degrees in each one, so you need to turn
180*6 = 1080 degrees arround that point to return to the
starting point. At the point z=1728, a path arround it goes
through two different spheres 360 degrees each, 720 in total.
At z=infinity the circles go through infinitely many spheres
and have infite length.

> I have the impression that you can't make a noncompact 3-manifold with
> constant positive curvature. Do you know if that's true or not? (I'm

[...]

I guess that can also be done in a similar way as above, but if
exceptional branching points are not allowed, you might be right.


Miguel A. Lerma


Ben Weiner

unread,
Jan 7, 1996, 3:00:00 AM1/7/96
to
mle...@arthur.ma.utexas.edu (Miguel Lerma) writes:

>By the way, I have always found intriguing the relation between
>the local structure of the universe (as a differenciable manifold)
>and its global topology. I think most of the time cosmologists make
>implicit assumptions about how they are related. In particular they
>estimate the size of the universe from its local curvature. However,
>it seems to me that they are different problems. I can imagine, for
>instance, manifolds of zero curvature and finite size, e.g. a plane
>torus.

Interestingly, universes with torus-like geometry (that is, periodic
in one or more spatial dimensions, often called "small universes")
can be ruled out by observations. The spatial periodicity introduces
a long-wavelength cutoff and this distorts the spectrum of the cosmic
microwave background radiation. I believe the limits from observations
are actually good enough now to rule out these small-universe models.

>Also I can conceive, say, infinite 2-manifolds with constant

>positive curvature. A sofisticated but interesting example ... [deleted]

>In short, I do not think that the local structure of the universe
>allows us to get conclusions about its global structure without
>additional assumptions.

Well, quite probably, but the limits on alternative (non-Friedmann)
universes may be quite strict. If one can cook up a model which
fits all the observations and makes no predictions differently from
a Friedmann model, that is of theoretical interest but may not be
of any practical interest to astronomers, especially observers.
Them's the breaks.


--
NO STEP

Robert Israel

unread,
Jan 7, 1996, 3:00:00 AM1/7/96
to
In article <4cnqv0$4...@electron.rutgers.edu>,
Ben Weiner <bwe...@electron.rutgers.edu> wrote:
> Outside the horizon, the universe could do just
>about anything it wants and we'd have no way of knowing. There could
>be a domain wall sweeping everything into oblivion heading for us at
>the speed of light, ready to come through the horizon tomorrow, and we
>wouldn't know - nor should we care much. It wouldn't get here for
>approximately a bazillion years anyway.

??? If it's heading directly for us at the speed of light, we wouldn't
know about it until it was already here.
--
Robert Israel isr...@math.ubc.ca
Department of Mathematics (604) 822-3629
University of British Columbia fax 822-6074
Vancouver, BC, Canada V6T 1Y4

Ben Weiner

unread,
Jan 7, 1996, 3:00:00 AM1/7/96
to
I wrote:

>Interestingly, universes with torus-like geometry (that is, periodic
>in one or more spatial dimensions, often called "small universes")
>can be ruled out by observations. The spatial periodicity introduces
>a long-wavelength cutoff and this distorts the spectrum of the cosmic
>microwave background radiation. I believe the limits from observations
>are actually good enough now to rule out these small-universe models.

That is, to rule out models in which the periodicity is significantly
smaller than the horizon size (the size of the observable universe),
as Ted Bunn said. Outside the horizon, the universe could do just


about anything it wants and we'd have no way of knowing. There could
be a domain wall sweeping everything into oblivion heading for us at
the speed of light, ready to come through the horizon tomorrow, and we
wouldn't know - nor should we care much. It wouldn't get here for
approximately a bazillion years anyway.

This is what I alluded to when I said that nonstandard topologies
may be theoretically amusing but not of much practical interest.

I believe de Oliveira-Costa & Smoot (1995, Ap.J. 448, 477 - "Constraints
on the Topology of the Universe from the 2 Year COBE Data") discusses
the present limits. Oh, I found another abstract:

Stevens, Scott & Silk, 1993, Phys Rev Lett 71, 20 -
"Microwave background anisotropy in a toroidal universe."

Abstract: Large-scale cosmic microwave background temperature
fluctuations are calculated for a universe with the topology of a
3-torus. In such a universe only perturbations which are harmonics of
the fundamental mode are permitted. By comparison with data from the
Cosmic Background Explorer satellite, we find that the minimum
(comoving) scale of a cubic toroidal universe is 2400/h Mpc for an n =
1 inflationary model. This is approximately an order of magnitude
greater than previous limits and 80 percent of the horizon scale,
implying that a topologically 'small' universe is no longer an
interesting cosmological model.


--
"If current World Wide Web usage trends continue, as with prior Internet growth,
we project that the US economy will collapse on June 10, 1998, as the rate of
white collar workers going pointy-clicky pointy-clicky all day goes asymptotic."
--- President's Council of Economic Advisors report, 12/1/95 (classified)

Edward Green

unread,
Jan 7, 1996, 3:00:00 AM1/7/96
to
'egr...@nyc.pipeline.com (Edward Green)' wrote:

>Now, to shift gears, I've been trying my hand as a pro-aether crank

What, no replies! Oh, I can see you shaking your heads sadly...

100 times: I will *not* write aether. I will *not* write aether...


I'd still like to know if the red-shift/temperature-shift equivalence in
black-body spectrums is some incredible cosmic coincidence, or the source
of deep insight. Does anybody have an opinion on that?

--

Ed Green
egr...@nyc.pipeline.com

Edward Green

unread,
Jan 7, 1996, 3:00:00 AM1/7/96
to
'b...@ipp-garching.mpg.de (Bruce Scott TOK )' wrote:

>It is a result of the fact that the blackbody spectrum is a function
>only of (h nu/k T) times a multiplier which depends only on T. (This is
>why the intensity of the background also decreases with expansion.)

Mathematically, yes. I inspected the equation. I was wondering what it
meant though. You could argue that the mathematics is the end of the
thing, but I don't find that very satisfying.

Gentlemen, a little hand-waving music, please.

--

Ed Green
egr...@nyc.pipeline.com

Bruce Scott TOK

unread,
Jan 7, 1996, 3:00:00 AM1/7/96
to
Edward Green (egr...@nyc.pipeline.com) wrote:

: I'd still like to know if the red-shift/temperature-shift equivalence in


: black-body spectrums is some incredible cosmic coincidence, or the source
: of deep insight. Does anybody have an opinion on that?

It is a result of the fact that the blackbody spectrum is a function


only of (h nu/k T) times a multiplier which depends only on T. (This is
why the intensity of the background also decreases with expansion.)

--

Edward Green

unread,
Jan 7, 1996, 3:00:00 AM1/7/96
to
'mle...@arthur.ma.utexas.edu (Miguel Lerma)' wrote:

>Your remarks are perfectly sound. I was just playing around with
>"naive" models of the universe ... <cut> ...
>a model in which the universe is like a balloon growing
>from an initial point, and the radial coordinate would be the time...

Well, even on this "naive" level, surely you mean, would be "proportional
to" (a monotonically increasing function of) time. I mean, it's a habit
of some skeins of popular physics to try to make everything sound as
abstruse as possible, but on the level of this naive model, it is the
third spatial dimension, not time, that is "orthogonal" to visible space.
It just happens to be growing in time.

Of course, it sounds much cooler the other way :-)

--

Ed Green
egr...@nyc.pipeline.com

Ilja Schmelzer

unread,
Jan 8, 1996, 3:00:00 AM1/8/96
to
In article <4ck30f$b...@geraldo.cc.utexas.edu> mle...@arthur.ma.utexas.edu (Miguel Lerma) writes:

>I think Hawking
>suggested that the whole 4-dimensional manifold is curved, and that
>the Big Bang and the Big Cruch are just points like any other
>else except for the fact that in a certain system of cordinates
>all coordinates except one (i.e.: the three spacial coordinates
>but not time) converge there (something similar to what happens
>in the surface of the Earth with meridians and paralels).

1. It is not a suggestion of Hawking, but a solution of the Einstein
equations found by Friedman (~1922), which is now the "standard model"
of the evolution.

2. The "big bang" is not a point like the other, but a singularity,
that means in this point it is not a solution of the Einstein
equations.


>If that
>is so, then the "center" of the universe can be conceived only
>with help of a fith dimension, so than the universe is a 4-sphere
>inmersed in a 5-dimencional Euclidean space.

3. There is no center of the universe in these solutions.

4. "Curvature" is a purely mathematical notion. Mathematicians have a
strange habit to use the same name for completely different things, if
only they may be described with the same formulas. Nice and simple for
mathematicians, but often confusing for laymen.

If you measure the distances in a room using a ruler, without
considering any relativistic effects, the "geometry" you obtain is
also "curved" in the mathematical sense. Simply because the
temperature has an influence on the length of your ruler.

"Curved space-time" does not mean that the space-time is really a
curved surface in something higher-dimensional. It also simply
describes the fact that our "rulers" are influenced by the
gravitational field. Nothing more.

Ilja
--
My concept for the quantization of gravity: ~/PG/
--------------------------------------------------------------------------
Ilja Schmelzer, D-10178 Berlin, Keibelstr. 38, <schm...@wias-berlin.de>
my WWW ~ page: http://www.wias-berlin.de/~schmelze
--------------------------------------------------------------------------

Hannu Poropudas

unread,
Jan 8, 1996, 3:00:00 AM1/8/96
to
In article <4ck30f$b...@geraldo.cc.utexas.edu>
which is dated 5 jan 1996 20:51:27 GMT
mle...@arthur.ma.utexas.edu (Miguel Lerma) wrote:
(I copied article by hand below)

>I guess the closest to this idea is what Richard A. Schumacher says
>in his post: the center is 15 million years in the past (Big Bang).

>But I think this assumes that only the 3-space is curved, and that


>the whole 4-dimensional manifold made up with the whole past, present

>and future history of the universe is Euclidean. It is still possible

>that the universe is even more complicated than that. I think Hawking


>suggested that the whole 4-dimensional manifold is curved, and that

>the Big bang and Big Crunch are just points like any other


>else except for the fact that in a certain system of cordinates
>all coordinates except one (i.e.: the three spacial coordinates

>but not time) converge there (something similar what happens
>in the surface of the earth with meridians and paralels). If that


>is so, then the "center" of the universe can be conceived only

>with help of a fifth dimension, so than the universe is a 4-sphere
>inmersed in a 5-dimensional Euclidean space.

I don't know if above is correct but when I asked Hanna-Maria question
about center of space in due time she answered that the Highest could
not live without the 'big ball in center of the space'. And I
understood that the Universe could not exist if that 'big ball
in center of the space' (= 'big strange crystal', which mass is
about +10^50 kg, and which resists contraction of the space around
it inside 'event horizon', in center of the space) would not exist.

Then I asked her that is center of space in center of Virgo Super
Cluster in center of M87 and she answered yes it is there.

If there does not exist center of the space, then the Universe
could not exist.?

Perhaps it is so that expansion of Universe needs those 'strange
crystals' as a 'springboard' for its expansion.?

Please take a look those README-articles in my directory in anonymous
WWW.FUNET.FI (or FTP.FUNET.FI) pub/doc/misc/HannuPoropudas.

Edward Green

unread,
Jan 8, 1996, 3:00:00 AM1/8/96
to
'schm...@fermi.wias-berlin.de (Ilja Schmelzer)' wrote:

>"Curved space-time" does not mean that the space-time is really a
>curved surface in something higher-dimensional. It also simply
>describes the fact that our "rulers" are influenced by the
>gravitational field. Nothing more.

You speak with great confidence about the unknowable.

--

Ed Green
egr...@nyc.pipeline.com

Bruce Scott TOK

unread,
Jan 8, 1996, 3:00:00 AM1/8/96
to
Edward Green (egr...@nyc.pipeline.com) wrote:
: 'b...@ipp-garching.mpg.de (Bruce Scott TOK )' wrote:
:
: >It is a result of the fact that the blackbody spectrum is a function
: >only of (h nu/k T) times a multiplier which depends only on T. (This is
: >why the intensity of the background also decreases with expansion.)
:
: Mathematically, yes. I inspected the equation. I was wondering what it

: meant though. You could argue that the mathematics is the end of the
: thing, but I don't find that very satisfying.
:
: Gentlemen, a little hand-waving music, please.

It is a matter of statistics. There are very many photons in the bath,
so the distribution of their energies follows what quantum statistics
allows as the maximum entropy (for nonrelativistic classical particles,
the distribution is a Maxwellian, but quantum effects change this to the
blackbody distribution formula for integer-spin particles).

In statistical physics, the distribution has a given shape which is
parameterised by the temperature (alternatively, you specify the total
energy of the total number of particles, and the temperature falls out
as a defined quantity, in terms of this parameterisation). For a
Maxwellian the average energy per particle works out to (3/2) kT; for a
photon bath it has some other numerical multiplier. But the
distribution is a function only of energy per particle divided by kT
since that is the definition of temperature.

If this sounds circular, it is because there are actually three
quantities (number of particles, total energy, temperature), and you
choose two to get the third.

The thing you want to read about if you want to explore further is
thermodynamics and entropy, not quantum mechanics or electrodynamics,
since the key to the whole mess is entropy.

Miguel Lerma

unread,
Jan 9, 1996, 3:00:00 AM1/9/96
to
Ilja Schmelzer (schm...@fermi.wias-berlin.de) wrote:

: In article <4ck30f$b...@geraldo.cc.utexas.edu> mle...@arthur.ma.utexas.edu (Miguel Lerma) writes:

> >I think Hawking
> >suggested that the whole 4-dimensional manifold is curved, and that

> >the Big Bang and the Big Cruch are just points like any other

> >else except for the fact that in a certain system of cordinates
> >all coordinates except one (i.e.: the three spacial coordinates

> >but not time) converge there (something similar to what happens
> >in the surface of the Earth with meridians and paralels).

> 1. It is not a suggestion of Hawking, but a solution of the Einstein
> equations found by Friedman (~1922), which is now the "standard model"
> of the evolution.
>
> 2. The "big bang" is not a point like the other, but a singularity,
> that means in this point it is not a solution of the Einstein
> equations.

Here is where Hawking's claim comes. It seems that quantum
considerations allows to conclude that t=0 it is not really a
singularity, but a "normal" point. I mention it because it
makes more natural the comparison with the surface of Earth.

[...]


> "Curved space-time" does not mean that the space-time is really a
> curved surface in something higher-dimensional. It also simply
> describes the fact that our "rulers" are influenced by the
> gravitational field. Nothing more.

I know that. I was just trying to imagine an scenary in which
the original question could make some sense, and a possibility
is to think of space-time as a submanifold of a higher dimensional
Euclidean space, since only in an Euclidean space the computation
of a "center" makes sense. But physical experience does not allow us
to go beyond intrinsic geometry.


Miguel A. Lerma


Holger Bruns

unread,
Jan 9, 1996, 3:00:00 AM1/9/96
to
David Jacob <dja...@toto.csustan.edu> writes:

>In the beginning, the Universe issued from a single point, and was
>composed of a at least 11 dimensions. Then the universe became unstable,
>and all but 3 of the dimensions collapsed, releasing tremendous energies,
>and causing the center of the universe to actualy detach and become a
>free wandering anomolie. The Center of the Universe has been wandering
>around ever since, and has been sought after by the various civilizations
>and species of the galaxy since the dawn of life. It is said that
>whomsoever possesses the true center of the universe possesses infinite
>power to reshape reality and bend others to his will...

Thank you for pointing that out. It sounds like Star Trek....

Holger


Holger Bruns

unread,
Jan 9, 1996, 3:00:00 AM1/9/96
to
In article <4cif07$f...@lmfpub.lmf.ericsson.se> Hannu.P...@ericsson.fi (Hannu Poropudas) writes:

>In article <4chedk$i...@geraldo.cc.utexas.edu>
>which is dated 4 Jan 1996 20:47:48 GMT
>mle...@arthur.ma.utexas.edu (Miguel Lerma) wrote:

>>Hannu Poropudas (hannu.P...@ericsson.fi) wrote:
>>
>>: By the way do you know where in our Universe is the center of
>>: the space (I mean the mass center of all galaxies).?
>>
>>Probably it is indefined. It is like asking about a center of
>>masses of the continents in the surface of the Earth.
>>

>Earth have a center, but it is outside of Earth's surface.

As far as I know, a center of the universe does not exist. The universe has no
beginning and no end in time and space.

Holger


john baez

unread,
Jan 9, 1996, 3:00:00 AM1/9/96
to
In article <4csjnr$r...@geraldo.cc.utexas.edu> mle...@arthur.ma.utexas.edu (Miguel Lerma) writes:

>Here is where Hawking's claim comes. It seems that quantum
>considerations allows to conclude that t=0 it is not really a
>singularity, but a "normal" point.

Well, sort of, but it's rather subtler than you might expect.

Edward Green

unread,
Jan 9, 1996, 3:00:00 AM1/9/96
to
'b...@ipp-garching.mpg.de (Bruce Scott TOK )' wrote:
>
>In statistical physics, the distribution has a given shape which is
>parameterised by the temperature (alternatively, you specify the total
>energy of the total number of particles, and the temperature falls out
>as a defined quantity, in terms of this parameterisation). For a
>Maxwellian the average energy per particle works out to (3/2) kT; for a
>photon bath it has some other numerical multiplier. But the
>distribution is a function only of energy per particle divided by kT
>since that is the definition of temperature.

Well, I've been thinking along these lines, and reached a similar
conclusion (I really didn't know -- I don't ask (too many) Socratic
questions :-)

Now, and here comes the aha! reaction, I think the key is...

The distribution of modes of the EM field is self-similar under shifts of
the energy scale.

This means a mapping of a bath of light quanta to itself where each photon
is reduced in energy by a factor k corresponds to another maximum entropy
distribution with total energy reduced by a factor k.

Another symmetry principle? The EM field is invariant under simultanteous
scaling of space and shift in the energy scale?

This still leaves me with the nagging question, is *this* a cosmic
coincidence, or a key to something deeper.

This time I don't really expect an answer.

--

Ed Green
egr...@nyc.pipeline.com

Ilja Schmelzer

unread,
Jan 9, 1996, 3:00:00 AM1/9/96
to
In article <4csjnr$r...@geraldo.cc.utexas.edu> mle...@arthur.ma.utexas.edu (Miguel Lerma) writes:

>> 2. The "big bang" is not a point like the other, but a singularity,
>> that means in this point it is not a solution of the Einstein
>> equations.

>Here is where Hawking's claim comes. It seems that quantum

>considerations allows to conclude that t=0 it is not really a

>singularity, but a "normal" point. I mention it because it
>makes more natural the comparison with the surface of Earth.

It is clear that quantum theory has to say something about this
point. It may be that this point becomes a normal point. Everything
may be :-). But it is not a "conclusion" at the current moment. Many
other things may happen with this point.

In my concept of quantum gravity, this point is simply not part of the
complete solution (see ~/PG/bibBang.html).

Edward Green

unread,
Jan 9, 1996, 3:00:00 AM1/9/96
to
'schm...@fermi.wias-berlin.de (Ilja Schmelzer)' wrote:

>>>"Curved space-time" does not mean that the space-time is really a
>>>curved surface in something higher-dimensional. It also simply
>>>describes the fact that our "rulers" are influenced by the
>>>gravitational field. Nothing more.
>
>>You speak with great confidence about the unknowable.
>
>The reverse is true.
>
>That the rulers are influenced by the gravitational field is the only
>thing we _know_. We can measure it and speak with great confidence
>about this effect.
>
>The other things which people usually associate with the notion of
>"curved space-time" are the "unknowable". Nonetheless many (other)
>people speak with great confidence about them.
>

I apologize sincerely for the tone of my answer.

But, to repeat something I said in reply to two other people who took me
to task for this by email, I assume this would rule out certain topologies
for the universe. In other words, you can't have a closed universe
embedded in 3 dimensional space. Since knowledgeable people seriously
entertain these topologies, and since so far as I know the language of
them is GR, there are at least alternate points of view entertained by
intelligent people.

You do state your position strongly.

--

Ed Green
egr...@nyc.pipeline.com

Bruce Scott TOK

unread,
Jan 9, 1996, 3:00:00 AM1/9/96
to
Edward Green (egr...@nyc.pipeline.com) wrote:

[...]

: The distribution of modes of the EM field is self-similar under shifts of
: the energy scale.

More or less what I said.

[...]

: This still leaves me with the nagging question, is *this* a cosmic


: coincidence, or a key to something deeper.

Just basic thermodynamics and the way temperature is defined. If you
want, the self-similarity is a consequence of the fact that the energy
of a photon is inversely proportional to wavelength, or...

its energy is its momentum times c, or...

photons are massless!


Let's see if Vergon jumps in :-)

Ilja Schmelzer

unread,
Jan 9, 1996, 3:00:00 AM1/9/96
to
In article <4cs0vb$k...@pipe10.nyc.pipeline.com> egr...@nyc.pipeline.com (Edward Green) writes:

>>"Curved space-time" does not mean that the space-time is really a
>>curved surface in something higher-dimensional. It also simply
>>describes the fact that our "rulers" are influenced by the
>>gravitational field. Nothing more.

>You speak with great confidence about the unknowable.

The reverse is true.

That the rulers are influenced by the gravitational field is the only
thing we _know_. We can measure it and speak with great confidence
about this effect.

The other things which people usually associate with the notion of
"curved space-time" are the "unknowable". Nonetheless many (other)
people speak with great confidence about them.

Ilja

john baez

unread,
Jan 9, 1996, 3:00:00 AM1/9/96
to
Edward Green (egr...@nyc.pipeline.com) wrote:

: Mathematically, yes. I inspected the equation. I was wondering what it
: meant though. You could argue that the mathematics is the end of the
: thing, but I don't find that very satisfying.

If you understand the equation *and its derivation*, you should be
satisfied, since that's the purest expression of the reasoning involved.
The mathematics is not in the equation, its in the derivation.

mikko....@vtt.fi

unread,
Jan 10, 1996, 3:00:00 AM1/10/96
to hol...@deep.hb.provi.de, Hannu.P...@ericsson.fi
hol...@deep.hb.provi.de (Holger Bruns) wrote:

>The universe has no beginning and no end in time and space.

For space, that seems to be true; if there is an edge of any
kind, it is so far that we can't see it. But for time, the
world looks different: it looks like it had existed only a finite
time.

-----------------------------------------------------------------
Mikko J. Levanto Tel. +358 81 551 2448
VTT Electronics Fax +358 81 551 2320
P.O.Box 1100 Internet: Mikko....@vtt.fi
FIN-90571 Oulu, Finland
--------------- VTT - Technical Research Centre of Finland ------

Richard A. Schumacher

unread,
Jan 10, 1996, 3:00:00 AM1/10/96
to
In <4d1mfc$i...@jobes.sierra.net> Bob Baker <rba...@foothill.net> writes:


>You big thinkers aren't listening to the question. Forget the math for a
>minute and listen. I want to know where the center of the universe is
>located. When I stand outside at night and look at the Southern Cross,
>or Ursa Major, or Polaris or whatever. I want to point to one of them
>and say, "So the Center of the Universe is that way." Can't one of you
>answer that question?


No, at least not with the kind of answer you want, because the question
is ill-formed. There is no point in the universe which could be called
the center any more than any other point. Now, do you want to know _why_
that is so?


Maybe we should go back to the old terminology used in popular
descriptions of cosmology before 1965 or so. Then the two possibilities
for the global structure or "shape" of the universe were described as
"infinite" and "finite but boundless". These may seem a bit mysterious,
but at least they don't encourage the reader to think erroneously of a
"center".


john baez

unread,
Jan 10, 1996, 3:00:00 AM1/10/96
to
In article <4ctt2p$g...@pipe10.nyc.pipeline.com> egr...@nyc.pipeline.com (Edward Green) writes:

>Another symmetry principle? The EM field is invariant under simultanteous
>scaling of space and shift in the energy scale?

>This still leaves me with the nagging question, is *this* a cosmic


>coincidence, or a key to something deeper.

>This time I don't really expect an answer.

Come, come: you expect far too little. As I noted before, rescaling
time is the same as rescaling temperature, quite generally. What's
special about the electromagnetic field is that it's "conformally
invariant". This implies that if you have any solution of Maxwell's
equations, and you simultaneously rescale space and time by the same
factor, you get a new solution.

It turns out that all the field equations of massless particles are
conformally invariant, but not the field equations of any massive
particles. This is easy to believe, because using hbar and c, units of
mass can be converted to units of inverse length. That's the idea
behind the Compton wavelength: any particle of a given mass has a
characteristic wavelength associated to it, which is proportional to one
over the mass.

So you can make light that's as big (radio waves) or small (gamma rays)
as you like, but you can't make atoms of arbitrary size.

Here's something I wrote about this once upon a time. For more,
check out

http://math.ucr.edu/home/baez/lengths.html

-------------------------------------------------------------------------
The Compton wavelength of the electron

The Compton wavelength of a particle, roughly speaking, is the length
scale at which relativistic quantum field theory becomes crucial for its
accurate description. A simple way to think of it is this.
Trying to localize an electron to within less than its Compton
wavelength makes its momentum so uncertain that it can have an energy
large enough to make an extra electron-positron pair! This is the
length scale at which quantum field theory, which describes particle
creation, becomes REALLY important for describing electrons. The
Compton wavelength of the electron is the characteristic length scale of
QED (quantum electrodynamics).

It's easy to guess how big the Compton wavelength is using the
knowledge that it depends only on the mass of the electron, relativity
and quantum mechanics. Mass has dimension M. Length has dimension L.
Times has dimension T. In relativity we have a constant, the speed of light,
with dimensions L/T, and in quantum mechanics we have a constant,
Planck's constant, with dimensions ML^2/T = energy times time =
momentum times position. These two constants enable us to express units
of mass in terms of dimensions of inverse length. I.e.:

M = (ML^2/T)(T/L)1/L = hbar/c 1/L.

So in particular the Compton wavelength should be about

L_{compton} = hbar/mc.

This is about 3 times 10^{-13} meters if I haven't gotten it screwed up.

We can also derive the Compton wavelength in a slightly more
enlightening manner as follows. The energy of an electron at rest is

mc^2

as someone once noted. Say we try to confine an electron in a region
of size L. Then the uncertainty principle says that its momentum
cannot only be known up to an error Delta p, where

L Delta p >= hbar.

If we make L small enough Delta p will be so big the electron may have
a kinetic energy bigger than twice the rest energy mc^2. This would be
enough to form an electron-positron pair! This effect -- the creation
of new particles while trying to determine the position of old ones --
will kick at the Compton wavelength

L_{compton} p = hbar 5)

where here p is the momentum at which the kinetic energy of an
electron is about mc^2 (forgetting the factor of 2, naturally).
Recall the souped-up version of E = mc^2 good for moving particles:

E^2 = m^2c^4 + p^2c^2,

and set the "kinetic energy squared" p^2c^2 equal to the rest energy
squared, (mc^2)^2. We get

p^2c^2 = m^2c^4 or p = mc!

So by 5) we have

L_{compton} = hbar/mc

as before. Again, a mass scale sets a length scale.

I strongly advise the reader to take the ratio of the Bohr radius by
the Compton wavelength to see how much bigger the atomic length scale
is than the length scale at which quantum field theory becomes really
important. It's about 137 if I didn't screw up. Of course this is a famous
number, one over the fine structure constant. I let the reader work out
what it is in terms of hbar, e, and c. It's good to keep this in mind
for a gut-level understanding of microphysics: quantum field theory
effects start really mattering for electrons on a distance scale 1/137
the size of the hydrogen atom. This is why people were able to notice
these field-theoretic effects and develop QED not too long after they
came up with a quantum-mechanical description of the hydrogen atom.

Matt McIrvin

unread,
Jan 10, 1996, 3:00:00 AM1/10/96
to
In article <4d119s$7...@guitar.ucr.edu>, john baez <ba...@guitar.ucr.edu> wrote:
>
>So in particular the Compton wavelength should be about
>
>L_{compton} = hbar/mc.
>
>This is about 3 times 10^{-13} meters if I haven't gotten it screwed up.

I think that historically, the Compton wavelength was defined as h/mc.
So what you give here is often called "the Compton wavelength over
2*pi." The h/mc definition makes more sense in terms of Compton's
original work on x-ray scattering off of electrons.

I know, it annoys me too. Your version is the more useful number;
for instance, it's the thing that has that cool 1/137 ratio with the
Bohr radius.

In the other direction, it has a 1/137 ratio with the notorious
"classical radius of the electron"... give or take a 2*pi, maybe; I've
forgotten utterly... and as Feynman once said, "If you don't know
where the 2*pi's go, you don't know nothing"....
--
Matt 01234567 <-- Indent-o-Meter
McIrvin ^ Indentation will soon be too cheap to meter!

Edward Green

unread,
Jan 10, 1996, 3:00:00 AM1/10/96
to
'ba...@guitar.ucr.edu (john baez)' wrote:

>
>If you understand the equation *and its derivation*, you should be
>satisfied, since that's the purest expression of the reasoning involved.
>The mathematics is not in the equation, its in the derivation.

I'll buy that. But I want a substantial insider discount.

I think it is possible to derive an equation, and understand every step of
the derivation, but still miss something deep or beautiful that the
equation is telling us.

But I agree you will probably understand more about it than just by staring
at it.... I mean "inspecting" it.

--

Ed Green
egr...@nyc.pipeline.com

john baez

unread,
Jan 10, 1996, 3:00:00 AM1/10/96
to

>That the rulers are influenced by the gravitational field is the only
>thing we _know_. We can measure it and speak with great confidence
>about this effect.
>
>The other things which people usually associate with the notion of
>"curved space-time" are the "unknowable". Nonetheless many (other)
>people speak with great confidence about them.

There's nothing wrong with speaking about the unknowable. You can say
whatever you want about it, since we'll never know if you're right. The
thing to keep in mind, though, when speaking about the unknowable, is
that you shouldn't get upset if someone else says the opposite of what
you do! It's perfectly fine for them to say whatever they want, too.

That's why it's perfectly acceptable to speak of "curved spacetime" and
so on as is usually done in general relativity, and it's also perfectly
acceptable to say that spacetime is not curved and that it's just the
rulers that get stretched.


Ilja Schmelzer

unread,
Jan 10, 1996, 3:00:00 AM1/10/96
to
In article <4cu1l9$q...@pipe10.nyc.pipeline.com> egr...@nyc.pipeline.com (Edward Green) writes:

>I apologize sincerely for the tone of my answer.

No reason. (My tone sometimes may be not better. I hate PC and prefer
a clear language which nonsense names nonsense. Feel free to use such
a language in your future responses to me.)

>But, to repeat something I said in reply to two other people who took me
>to task for this by email, I assume this would rule out certain topologies
>for the universe.

Yes. All nontrivial topologies. This seems to be the only difference
between GR and the variant of GR I propose with absolute time
(~/PG/PG.html).

>In other words, you can't have a closed universe
>embedded in 3 dimensional space.

But instead of the full closed universe you may use only a part. It is
hard to distinguish, to prove by observation that the topology is
really nontrivial even if it really is. It would be nice for me to
make the statement "My theory predicts that the universe is flat"
which seems to be in agreement with experiment, but it seems to be
false. All I can say about the "closed universe" is that it will be
not homogeneous (it has a center) from point of view of my theory.

>Since knowledgeable people seriously
>entertain these topologies, and since so far as I know the language of
>them is GR, there are at least alternate points of view entertained by
>intelligent people.

Yes. They are alternate points of view.

>You do state your position strongly.

A strongly state that my variant of GR (a different theory, a little
bit stronger than usual GR because it makes additional predictions
about the topology) is also an alternate point of view and in no
contradiction to observations. (Without necessity to make assumptions
about the intelligence of the person(s) entertaining this position :-)

Based on this idea, I strongly oppose every statement which excludes
this alternative without reason. This includes any statement of the
type "there is no absolute time". Until nontrivial topologies are
really observed in experiments, my variant (which has an absolute
time) is as good as the standard variant in explaining
experiments. (Even better, because it explains why we don't observe
nontrivial topologies. Remark they are based on the same equations -
the Einstein equations. See ~/PG/PGvsGR.html for details.) I do not
name "false" statements like "we don't know if there is an absolute
time" which are open for above variants.

Thus, I have a much weaker position about which variant is "true" (I
think only quantum gravity will show which is better, but I think also
that I have a lot of reasonable arguments in favour of absolute time),
but a strong position that it may be true according to the current
experiments.

john baez

unread,
Jan 10, 1996, 3:00:00 AM1/10/96
to
In article <4cp7jr$f...@pipe11.nyc.pipeline.com> egr...@nyc.pipeline.com (Edward Green) writes:

>I'd still like to know if the red-shift/temperature-shift equivalence in
>black-body spectrums is some incredible cosmic coincidence, or the source
>of deep insight. Does anybody have an opinion on that?

We should immediately assume that it's not a coincidence, since there
are very few coincidences of this sort. The question is then simply
whether it's a "deep" fact or a "shallow" one.

Actually, while not especially deep itself, it's a spinoff of the deep
fact that imaginary time and inverse temperature are two ways of talking
about the same thing!

More precisely, we should always think of

sqrt(-1) t/hbar,

where t is time and hbar is Planck's constant, as just another word for

1/kT,

where T is temperature and k is Boltzmann's constant. The point is that
time evolution is given (in quantum mechanics) by

exp(-itH/hbar)

where H is the Hamiltonian or energy operator, while the equilibrium
state in (quantum) statistical mechanics is given by

exp(-H/kT).

A similar relationship holds in classical mechanics, but as usual
classical mechanics is more complicated than quantum mechanics, so
I will only discuss the quantum case. (The real world is quantum, too,
so if we are going to talk about just one case let's talk about the real
one.)

This relationship is the basis of all sorts of wonderful things; it
means, for example, that quantum field theory and quantum statistical
mechanics are mathematically isomorphic. This is a very practical
isomorphism that physicists use all the time. It's also the basis of
Hawking's work on "imaginary time" and the statistical mechanics of
black holes.

In the present case, the point is that redshifts are due to *time
dilation*: the atoms wiggling in the receding star appear to be moving
slower to us. By the above, *time dilation* is equivalent to
"temperature dilation": the light from the receding star appears cooler
to us.

Bob Baker

unread,
Jan 11, 1996, 3:00:00 AM1/11/96
to
schm...@fermi.wias-berlin.de (Ilja Schmelzer) wrote:
>In article <4csjnr$r...@geraldo.cc.utexas.edu> mle...@arthur.ma.utexas.edu (Miguel Lerma) writes:
>
>>> 2. The "big bang" is not a point like the other, but a singularity,
>>> that means in this point it is not a solution of the Einstein
>>> equations.
>
>>Here is where Hawking's claim comes. It seems that quantum
>>considerations allows to conclude that t=0 it is not really a
>>singularity, but a "normal" point. I mention it because it
>>makes more natural the comparison with the surface of Earth.
>
>It is clear that quantum theory has to say something about this
>point. It may be that this point becomes a normal point. Everything
>may be :-). But it is not a "conclusion" at the current moment. Many
>other things may happen with this point.
>
>In my concept of quantum gravity, this point is simply not part of the
>complete solution (see ~/PG/bibBang.html).
>
>Ilja
>
>--
>My concept for the quantization of gravity: ~/PG/
>--------------------------------------------------------------------------
>Ilja Schmelzer, D-10178 Berlin, Keibelstr. 38, <schm...@wias-berlin.de>
>my WWW ~ page: http://www.wias-berlin.de/~schmelze
>--------------------------------------------------------------------------

You big thinkers aren't listening to the question. Forget the math for a

Kevin Johnson

unread,
Jan 11, 1996, 3:00:00 AM1/11/96
to
Tom Gossman (gos...@hcob.wmich.edu) wrote:
> In article <4d24ih$7...@agate.berkeley.edu>,
> t...@physics12.Berkeley.EDU (Emory F. Bunn) wrote:
> >
> >Or maybe you aren't listening to the answers. The answer, as several
> >of us have said, is that as far as we know there is no such place.
> >
> In other words, it isn't that there is no such place. Its just that they
> can't answer your question. The original question is a fair one, however.

Is this a fair question? WRT the location of our solar system right now,
what is a good estimation of the location of the origin of the Universe
when it first formed? No answer?

--
_____ _____ _____ | "The problem of the | Kevin Johnson
\ \/ /\/ /\ (R) | outsider is that he sees | kevin....@waii.com
\ / / / / \ | too deeply and too much, |_______________________
\ / / / / \ | and what he sees is |
\____/\/____/\_____\ | essentially chaos." | My company doesn't
| -- The Outsider, | agree with me about
WESTERN GEOPHYSICAL | Colin Wilson | anything.

Tom Gossman

unread,
Jan 11, 1996, 3:00:00 AM1/11/96
to
In article <4d24ih$7...@agate.berkeley.edu>,
t...@physics12.Berkeley.EDU (Emory F. Bunn) wrote:
>
>Or maybe you aren't listening to the answers. The answer, as several
>of us have said, is that as far as we know there is no such place.
>
In other words, it isn't that there is no such place. Its just that they
can't answer your question. The original question is a fair one, however.

Thomas L Gossman (Tom) thomas....@wmich.edu
FCL/Haworth College of Business Phone: 616-387-5524
Western Michigan University FAX: 616-387-5710
Kalamazoo MI 49008-3811
"Outside of a dog, a book is man's best friend. Inside of a dog,
its too dark to read." --Groucho Marx

Richard A. Schumacher

unread,
Jan 11, 1996, 3:00:00 AM1/11/96
to
In <4d3vbv$5...@henge2.henge.com> gos...@hcob.wmich.edu (Tom Gossman) writes:

>An assertion that "it is currently believed (by most cosmologists)" and
>another assertion that the question is just plain nonsense make me very
>suspicious.

And yet this is the case. One should always be prepared to suspect that
one does not know the whole story :->

>It seems logical that when we had the Big Bang, and space was
>created forming out in all directions, it all began at some point which is
>still there in that created space. Of course science can be contrary to
>common logic, but so far I have seen noting to indicate that there is any
>accpted theory (beyond assertions) that such is the case here.

Accepted theories are mostly similar to the Robertson-Walker metric.
The universe is either infinite and will expand forever, or is
finite _but with no boundary_ and will one day collapse on itself
again. You can read much more about the current state of affairs
in "In Search of the Big Bang" by John Gribben, and "The First
Three Minutes" by Steven Weinberg. They explain what we think we
know about the universe, and why we think we know it.

>As to the questions being nonsense, I disagree. It is a perfectly rational
>question, unlike a question about the color of time, etc.

They're both valid in the sense that asking them allows one
to clear up some evident misconceptions in the questioner
(time has no color, the universe need not have a center).

Edward Green

unread,
Jan 11, 1996, 3:00:00 AM1/11/96
to
'schu...@convex.com (Richard A. Schumacher)' wrote:

>No, at least not with the kind of answer you want, because the question
>is ill-formed. There is no point in the universe which could be called
>the center any more than any other point. Now, do you want to know >_why_
that is so?
>
>
>Maybe we should go back to the old terminology used in popular
>descriptions of cosmology before 1965 or so. Then the two possibilities
>for the global structure or "shape" of the universe were described as
>"infinite" and "finite but boundless". These may seem a bit mysterious,
>but at least they don't encourage the reader to think erroneously of a
>"center".

Not so much "ill-formed" as implicitly assuming some other topology than
the ones you mentioned. Acknowledging Schmelzer's position, we can only
see a finite chunk of the universe, so we can not rule out "finite and
*bounded*. At least so far as I know.

I think we don't speak of this possibility very often because it would
require assuming some new physics at the "boundry". Since we cannot see
any boundry, the working assumption is, there isn't one.

--

Ed Green
egr...@nyc.pipeline.com

Ilja Schmelzer

unread,
Jan 11, 1996, 3:00:00 AM1/11/96
to
In article <4d278m$m...@pipe9.nyc.pipeline.com> egr...@nyc.pipeline.com (Edward Green) writes:

>I don't understand the concept "inhomogeneous closed universe". You say it
>has a center. Does it have a boundry? You didn't contradict me when I
>said your universe is embedded in 3-space. I am hard-pressed to imagine
>such a universe being "closed"; but maybe I am thinking of "compact".

You have a sphere. Put it on a plane. Mark the upper point (North
pole). Now consider lines from the North pole to any point on the
plane. It intersects the sphere in one point.

This defines a homeomorphism of your sphere (closed universe) without
a single point - the North pole - and that's why no longer closed, on
the plane (our assumed in PG flat space).

But the metric of the sphere has a symmetry - of rotations. The plane
has also a symmetry. They are different. Thus, the symmetry group of
the image of this homeomorphism will be smaller - the intersection
between the two symmetry groups. There will be no translational
symmetry for this solution. There is one special point - the South
pole, which may be considered as the center of this solution. No
boundary.

How to distinguish these two interpretations by measurement I don't
know. We cannot prove by experiment that all points are really
present.

john baez

unread,
Jan 11, 1996, 3:00:00 AM1/11/96
to
In article <4d1hub$g...@decaxp.harvard.edu> mci...@fas.harvard.edu (Matt McIrvin) writes:
>In article <4d119s$7...@guitar.ucr.edu>, john baez <ba...@guitar.ucr.edu> wrote:

>>So in particular the Compton wavelength should be about

>>L_{compton} = hbar/mc.

>>This is about 3 times 10^{-13} meters if I haven't gotten it screwed up.

>I think that historically, the Compton wavelength was defined as h/mc.


>So what you give here is often called "the Compton wavelength over
>2*pi."

What a pain! I guess it's time to redefine the Compton wavelength.

>I know, it annoys me too. Your version is the more useful number;
>for instance, it's the thing that has that cool 1/137 ratio with the
>Bohr radius.

>In the other direction, it has a 1/137 ratio with the notorious

>"classical radius of the electron"... give or take a 2*pi, maybe...

Yes, in my article "Lengths", which the stuff above was quoting, I made
a big deal about the Bohr radius, the Compton wavelength, and the
classical electron radius, and how they differ by factors of 1/137. The
idea was that each 1/137 corresponds to an order of perturbation theory
in QED, so that the Bohr radius is roughly the scale at which quantum
mechanics starts mattering for the electron, while the Compton
wavelength is where first-order QED (e.g., particle/antiparticle
creation) starts mattering, while the classical electron radius is where
second-order QED (e.g, renormalization) starts mattering.

This is too nice to give up, so I'm gonna stick with my hbar's and forget
the h's.

Oz

unread,
Jan 11, 1996, 3:00:00 AM1/11/96
to
If you will forgive a small change of topic.

The Red Shift can be considered as either due to expansion
of space or a gravitational redshift, but apparently not
both simultaneously. Now Ted has previously described a
model (or co-ordinate system) based on co-moving
co-ordinates that I thought I had saved, but apparently not.
I believe the non-GR illustration was called the Milne
model.

Now this model suggested that the redshift would be the
expected view just because of SR effects (if I remember
correctly). However I suspect that a GR modelling would
perhaps get a slightly different picture.

So my question. Very simple and basic. When one says space
is expanding, do we really mean nothing more than galaxies
have motion away from each other due to essentially initial
velocities originating at the BB, or something more, well,
interesting. If this is the case why should not the
gravitational redshift be added in 'on top'. Since I expect
a GR answer, please bear in mind that my knowledge of it is
spectacularly rudimentary (non-existant?).

For example. If the expansion of space is indeed nothing
more than due to initial velocities (OK please define
velocity, what co-ordinate system etc etc, I know) then
explaining redshift by a 'stretching' of the light sounds
fun, but might be better described as the originating object
simply having, well, a high redshift due to it's velocity.
In which case I would still expect a gravitational redshift
on top.

Alternatively I am looking at it out of ignorance.
So ......


-------------------------------
'Oz "When I knew little, all was certain. The more I learnt,
the less sure I was. Is this the uncertainty principle?"

Edward Green

unread,
Jan 11, 1996, 3:00:00 AM1/11/96
to
On Jan 10, 1996 14:07:34 in article <Re: Center of Universe?>,
'schm...@fermi.wias-berlin.de (Ilja Schmelzer)' wrote:

>... my variant of GR (a different theory, a little
>bit stronger than usual GR because it makes additional predictions
>about the topology) is also an alternate point of view and in no
>contradiction to observations...
>
>Based on this idea, I strongly oppose every statement which excludes
>this alternative without reason. This includes any statement of the
>type "there is no absolute time". Until nontrivial topologies are
>really observed in experiments, my variant (which has an absolute
>time) is as good as the standard variant in explaining
>experiments. (Even better, because it explains why we don't observe
>nontrivial topologies. Remark they are based on the same equations -
>the Einstein equations. See ~/PG/PGvsGR.html for details.) I do not
>name "false" statements like "we don't know if there is an absolute
>time" which are open for above variants.
>
>Thus, I have a much weaker position about which variant is "true" (I
>think only quantum gravity will show which is better, but I think also
>that I have a lot of reasonable arguments in favour of absolute time),
>but a strong position that it may be true according to the current
>experiments.

Well, you have convinced me of your logical honesty with respect to the
known and the unknown, and so, not having any more incisive comments
regarding the strength of the evidence, I bow out...

Which would have been a good place to stop, except that I really want to
ask you about something :-) ...

>It would be nice for me to
>make the statement "My theory predicts that the universe is flat"
>which seems to be in agreement with experiment, but it seems to be
>false. All I can say about the "closed universe" is that it will be
>not homogeneous (it has a center) from point of view of my theory.

I don't understand the concept "inhomogeneous closed universe". You say it
has a center. Does it have a boundry? You didn't contradict me when I
said your universe is embedded in 3-space. I am hard-pressed to imagine
such a universe being "closed"; but maybe I am thinking of "compact".

--

Ed Green
egr...@nyc.pipeline.com

Edward Green

unread,
Jan 11, 1996, 3:00:00 AM1/11/96
to
'ba...@guitar.ucr.edu (john baez)' wrote:

>
>There's nothing wrong with speaking about the unknowable. You can say
>whatever you want about it, since we'll never know if you're right. The
>thing to keep in mind, though, when speaking about the unknowable, is
>that you shouldn't get upset if someone else says the opposite of what
>you do! It's perfectly fine for them to say whatever they want, too.

I like that. The "I'm ok, you're ok" philosophy of the unknowable.

But maybe, even if it makes us sound like pompous old pedants, it would
be nice to occasionally add an "As far as it seems to me" or "In my way of
looking at things". Something to avoid the appearance of rigid certainty.

--

Ed Green
egr...@nyc.pipeline.com

Ville Sinkko

unread,
Jan 11, 1996, 3:00:00 AM1/11/96
to
Hannu Poropudas (hannu.P...@ericsson.fi) wrote:

: By the way do you know where in our Universe is the center of
: the space (I mean the mass center of all galaxies).?

Just listen for the sound of the flutes... ;)

--
65
Zn
30


Miguel Lerma

unread,
Jan 11, 1996, 3:00:00 AM1/11/96
to
Mahipal Singh Virdy (vi...@pogo.den.mmc.com) wrote:
[...]
> So far you've been offered three answers:
> 1. There is no center (dominant answer)
> 2. The question is ill-posed (We're not sure what to say)
> 3. Each point anywhere at all is the "center"

Let me pose the subject in another way. Let's try the following
definition of "center" of the universe:

The mass center of the universe is the point of position
vector R such that:

(1/M) SUM (r_i - R) m_i = 0

where M is the total mass of the universe, m_i is the mass
of the i-th particle of the universe, r_i is a position vector
of that particle, and the sum goes throght all the particles in
the universe.

Posed that way, the problem looks like just solving an equation
with an unknown R, right? If the universe has a center, the equation
should have a unique solution. No center (answer "1" above) could mean
that the equation has no solution (although I think that many people
answering "1" actualy mean "2"). "Any point is the center"
(answer "3") means that the equation is indeterminate. But the
fact is that in the currently accepted models of the universe,
there are no such a thing as vectors "r_i", so the equation
cannot be even posed. The right answer is "2" (at least according
to current models), but we do know what to say: the question rests
on wrong assumptions. The universe lacks a center not because the
equation has no solutions, but because there is not even equation.


Miguel A. Lerma


Mahipal Singh Virdy

unread,
Jan 11, 1996, 3:00:00 AM1/11/96
to
In article <4d1mfc$i...@jobes.sierra.net>,

Bob Baker <rba...@foothill.net> wrote:
>
>You big thinkers aren't listening to the question. Forget the math for a
>minute and listen. I want to know where the center of the universe is
>located. When I stand outside at night and look at the Southern Cross,
>or Ursa Major, or Polaris or whatever. I want to point to one of them
>and say, "So the Center of the Universe is that way." Can't one of you
>answer that question?

I'm a mechanical engineering by the "training". Let me try answer.


So far you've been offered three answers:
1. There is no center (dominant answer)
2. The question is ill-posed (We're not sure what to say)
3. Each point anywhere at all is the "center"

The question of center is complicated because one has to define what
integer dimension (1,2,3,4, ..., Multi, ..., Inf) our Universe is.
That's a hotly debated issue and thus there can be no concensus among
the holders of the varying perspectives. You want to bypass all that
gibberish. I'm with you! ;-)

At any frozen time instant, the Universe is 3 spatial dimensions. Don't
wake up any of the string theorists because no one knows what plane
they're on at any given moment. 10, 26? Let's say space is just 3D and
time is frozen dead still. A Kodak Moment. Where's the center in this
sense?

You want the geometric center, the mass weighted center, the temperature
weighted center, the money weighted center, or ...? There are many
useful centers to define. Let's assume you mean the geometric center. In
this sense, think of a Cartesian Grid. Where's point (0,0,0) becomes
your question.

Space extends for infinty in all direction. At least, observably we here
on Earth can see equally far in all directions. According to many
models, this would be true no matter where you translate yourself in the
Universe. Of course, this is untestable and unprovable for all intents
and practical purposes.

To me, this means that the center is where ever you want it to be! It's
undefinable yet everywhere. The expanding surface of the baloon analogy
is used to make this point that all points are equivalent centers. I'm
sure this is *not* what you wanted to hear, read I mean.

Let me ask you, where's the center of one's "mind"? Does it bother you
in any significant way that a human can't exactly locate a center even
within the spatial confines of one's own self?

The Universe is a 4D object as a minimum. ... How about this, the center
of the Universe is precisely every place where you instanteously happen
not to be? ;-) That is if you can't *handle* the idea of a Universe
centered around you. Enjo(y).

[I could go on but I tire...]

Mahipal |meforce>
http://www.princeton.edu/~msvirdy/index.html

Miguel Lerma

unread,
Jan 11, 1996, 3:00:00 AM1/11/96
to
Tom Gossman (gos...@hcob.wmich.edu) wrote:
> An assertion that "it is currently believed (by most cosmologists)" and
> another assertion that the question is just plain nonsense make me very
> suspicious. It seems logical that when we had the Big Bang, and space was
> created forming out in all directions, it all began at some point which is
> still there in that created space. Of course science can be contrary to
> common logic, but so far I have seen noting to indicate that there is any
> accpted theory (beyond assertions) that such is the case here.

Well, sometimes science is contrary to a naive extrapolation of
common experience. For instance, the most natural extrapolation
of what we see arround us is to believe that the surface of the
Earth is flat, and perhaps either extended infinitely in all
directions, or ending somewhere. So it could seem natural to
ask where the border of the surface of the Earth is. However
it is currently believed (by most geographers) that the Earth
is not flat, but round like a sphere, so the question, even if
perfectly acceptable, actually does not make sense, because
it rests on a wrong assumption (i.e.: that the Earth is flat).

The idea of the space forming out in all directions from a
starting is another kind of missconception. The points of
the space have no individual indentity, so the question of
if some point is the one where the Big Bang started does
not make any sense either. When we talk of some place on
Earth, such as Paris, or the dinner room in my house, we
are not talking about space, but about material objects.
What remains is the object (Paris, a room, or whatever),
not the "space". Anyway, it reminds me of an old joke about
a man who had a stick that was too long, so he decided to
cut out its golden handle. When someone asked why he didn't
cut the stick at the other end, he answered: "at the other
end the stick was ok, it is at the handle end where it was
too long".

> As to the questions being nonsense, I disagree. It is a perfectly rational
> question, unlike a question about the color of time, etc.

I think it is a perfectly understable question, but if the
universe is as cosmologists currently believe, then that

question rests on wrong assumptions.

So you are claiming that the "color of time" is a nonsense?
How do you know? What do we know about the nature of time?


Miguel A. Lerma

P.S.: By the way, the members of the Flat Earth Society
claim that the Earth is flat, so the "theory"
that the Earth is round is not universally accepted.


Emory F. Bunn

unread,
Jan 11, 1996, 3:00:00 AM1/11/96
to
In article <4d1mfc$i...@jobes.sierra.net>,
Bob Baker <rba...@foothill.net> wrote:
:You big thinkers aren't listening to the question. Forget the math for a
:minute and listen. I want to know where the center of the universe is
:located. When I stand outside at night and look at the Southern Cross,
:or Ursa Major, or Polaris or whatever. I want to point to one of them
:and say, "So the Center of the Universe is that way." Can't one of you
:answer that question?

Or maybe you aren't listening to the answers. The answer, as several


of us have said, is that as far as we know there is no such place.

-Ted

Ilja Schmelzer

unread,
Jan 11, 1996, 3:00:00 AM1/11/96
to

>The Red Shift can be considered as either due to expansion
>of space or a gravitational redshift, but apparently not
>both simultaneously.

Why not?

>Now Ted has previously described a
>model (or co-ordinate system) based on co-moving
>co-ordinates that I thought I had saved, but apparently not.

The coordinates usually used in GR (I assume they are the coordinates
originally used by Friedman) are co-moving too.

>So my question. Very simple and basic. When one says space
>is expanding, do we really mean nothing more than galaxies
>have motion away from each other due to essentially initial
>velocities originating at the BB, or something more, well,
>interesting.

Even less. GR does not define "motion" or "velocities". GR defines
only that the "distance" between the points in the co-moving
coordinates is increasing (don't reply here, I know it's not
completely correct). The distance is what we measure using a ruler.

You can interprete this without any motion, simply assuming that there
is an ether with material properties which change in time, and this
ether has an influence on the electromagnetic field (and all other
fields). This influence makes all atoms, crystals and so on, including
our rulers, smaller.

You can assume there is no such ether, physics are the same, there is
one point at rest, and all other points are moving away from this
point.

GR does not tell us which of these two pictures or which probable
other picture is true. It defines only what we can measure with our
rulers. Not very much.

I have some hope that quantum gravity may allow to tell you more. If
I'm right, the first scenario is more probable. But, warning: this is
my own concept, not (yet?) accepted by the mainstream physics. But I
think I have good arguments.



>For example. If the expansion of space is indeed nothing
>more than due to initial velocities (OK please define
>velocity, what co-ordinate system etc etc, I know) then
>explaining redshift by a 'stretching' of the light sounds
>fun, but might be better described as the originating object
>simply having, well, a high redshift due to it's velocity.
>In which case I would still expect a gravitational redshift
>on top.

As GR does not tell us, if the galaxies are moving or not, is does not
tell us also which part of the redshift is caused by velocities and
which by gravity. Consider it as caused by the Doppler effect,
consider it as caused by gravity - your decision. You cannot prove it
in classical GR.

john baez

unread,
Jan 11, 1996, 3:00:00 AM1/11/96
to
In article <4d3gct$3...@mtnmath.com> pa...@mtnmath.com (Paul Budnik) writes:
>john baez (ba...@guitar.ucr.edu) wrote:

>: There's nothing wrong with speaking about the unknowable. You can say
>: whatever you want about it, since we'll never know if you're right.

>The funny thing about the unknowable is that it has a nasty habit of
>becoming known.

The unknown becomes known quite often, the unknowable less so. But
when it does, it wasn't really the unknowable after all, so you should
have been more careful about what you say about it.

Jeff Candy

unread,
Jan 11, 1996, 3:00:00 AM1/11/96
to
gos...@hcob.wmich.edu (Tom Gossman) wrote:
>In article <4d24ih$7...@agate.berkeley.edu>,
> t...@physics12.Berkeley.EDU (Emory F. Bunn) wrote:
>>
>>Or maybe you aren't listening to the answers. The answer, as several
>>of us have said, is that as far as we know there is no such place.
>>
>In other words, it isn't that there is no such place. Its just that they
>can't answer your question. The original question is a fair one, however.

Before you can ask, "what is the centre of the universe?",
you have to define the "centre of the universe". The geometry
of space-time is more complicated than the flat 3-D space which
forms the basis of most people's intuition, and an analogue to
the concept "centre" not immediately obvious. Define the concept
of centre and we'll tell you the answer, or point out how it can be
estimated. By one definition, the "centre" is at t=0+, for reasons
which have already been discussed.

This thread reminded me of the impossible-to-explain (sometimes)
question "what happened before the big bang?". Since the space-time
manifold is concieved to be a set open at t=0 (i.e., the universe
exists only for t>0), the question is ill-posed. This answer
typically illicits the response, "yeah, but what happened before
that???".

Jeff Candy ... man -- every man -- is an end
Analytic Theory Group in himself, not the means to the
JET Joint Undertaking ends of others ...
--- Ayn Rand


===============================================================================
The above article is the personal view of the poster and should not be
considered as an official comment from the JET Joint Undertaking
===============================================================================

Miguel Lerma

unread,
Jan 11, 1996, 3:00:00 AM1/11/96
to
Tom Gossman (gos...@hcob.wmich.edu) wrote:
> In article <4d24ih$7...@agate.berkeley.edu>,
> t...@physics12.Berkeley.EDU (Emory F. Bunn) wrote:
> >
> >Or maybe you aren't listening to the answers. The answer, as several
> >of us have said, is that as far as we know there is no such place.
> >
> In other words, it isn't that there is no such place. Its just that they
> can't answer your question. The original question is a fair one, however.

Not at all. If the universe is as it is currently believed to be
(by most cosmologists), then there is _no_ point that can be called
"the center of the universe" for any reasonable definition of the
term (some of us have been making a big effort of imagination trying
to make some sense of the question). It is like asking about the border
of a borderless surface (such as the surface of the Earth), or about the
"last" natural number, about the colour of time, or about the sex of the
last angel that can be put in the head of a pin. It is not that the answer
is unkonwn, or even that the question has no answer... in some way the
"question" is not even a question, but plain nonsense.

Note that I am not saying that such a question should not be asked.
It is perfectly right to honestly ask nonsense questions, but then
the right answers are going to deal rather with the nature of the
question and the hidden assumptions behind it.


Miguel A. Lerma

BEN

unread,
Jan 11, 1996, 3:00:00 AM1/11/96
to
In article <4d3925$d...@airgun.wg.waii.com>, ha9...@hocr2n23.london.waii.com (Kevin Johnson) writes:

> Is this a fair question? WRT the location of our solar system right now,
> what is a good estimation of the location of the origin of the Universe
> when it first formed?

Right here. Out there. Everywhere.

Ben


Richard A. Schumacher

unread,
Jan 11, 1996, 3:00:00 AM1/11/96
to

>>Or maybe you aren't listening to the answers. The answer, as several
>>of us have said, is that as far as we know there is no such place.
>>
>In other words, it isn't that there is no such place. Its just that they
>can't answer your question. The original question is a fair one, however.


No, it ain't. The only answer a cosmologist can give to the original
question is: to the best of human knowledge there is _no such place_.
The universe is either infinite, or finite but unbounded. Neither of
those possiblities has a center.

Clear enough?


john baez

unread,
Jan 11, 1996, 3:00:00 AM1/11/96
to

In <4d1mfc$i...@jobes.sierra.net> Bob Baker <rba...@foothill.net> writes:

>You big thinkers aren't listening to the question. Forget the math for a
>minute and listen. I want to know where the center of the universe is
>located. When I stand outside at night and look at the Southern Cross,
>or Ursa Major, or Polaris or whatever. I want to point to one of them
>and say, "So the Center of the Universe is that way." Can't one of you
>answer that question?

Sure, the center is in the direction of Antares.

Of course, this answer is complete bullshit, because the question is
meaningless. But if you want an answer anyway, this is as good as any.


Richard E. Carney

unread,
Jan 12, 1996, 3:00:00 AM1/12/96
to
t...@physics12.Berkeley.EDU (Emory F. Bunn) wrote:
>In article <4d1mfc$i...@jobes.sierra.net>,
>Bob Baker <rba...@foothill.net> wrote:
>:You big thinkers aren't listening to the question. Forget the math for a
>:minute and listen. I want to know where the center of the universe is
>:located. When I stand outside at night and look at the Southern Cross,
>:or Ursa Major, or Polaris or whatever. I want to point to one of them
>:and say, "So the Center of the Universe is that way." Can't one of you
>:answer that question?
>
>Or maybe you aren't listening to the answers. The answer, as several
>of us have said, is that as far as we know there is no such place.
>
>-Ted


The problem is that you have to stop thinking of the universe as a 'regular'
3-d object. If you take a sphere or a cube, you can explicitly say where the
centre is - in the case of the cube where the diagonals intersect. However,
thinking of the universe or space in such a way is wrong. Well not so much
incorrect as misleading, in the same way that considering atoms via the
billiard
ball model can be misleading. (c.f. QM)

I imagine that you consider yourself to be inside a large spherical
shell (the universe) and want to know which way to the centre of the shell.

Well look at it this way: If you are on a surface, the surface of the earth
say,
which direction will take you to the centre of the surface. Whichever direction
you choose will be the same and just move you to a different point. After you
have moved you are still no nearer or further away from your desired central
point. In this case if you keep going in the chosen direction for long enough
you will come back to where you started from.

This is how the earths surface can be finite in area but have no boundary. An
analogous version can be applied to the universe (although it is in a higher
order than a 2-d surface).

Hopefully this will have clarified things a little.


john baez

unread,
Jan 12, 1996, 3:00:00 AM1/12/96
to
In article <4d4cvf$8...@agate.berkeley.edu> t...@physics.berkeley.edu writes:

>I would tend to avoid saying that the question of where the center of
>the Universe is is "nonsense," since that's such a pejorative term,
>and you have to have studied the subject some before you understand
>why it's not a good question to ask. It turns out, however, that it's
>not a good question to ask. Specifically, it appears to be a question
>without an answer.

Ted Bunn is being levelheaded, evenhanded and fair here, as usual. In
more peevish responses, various people (including me) have been saying
that the question is "meaningless", "nonsense", or various other nasty
things.

The point is this: there are lots of questions in physics that start out
sounding like very interesting, important questions. Sometimes,
however, it turns out that --- as far as we can tell! --- they have no
answer. The reason is that the universe just isn't built in the way
that the question implicitly presupposed. For example, the question
of how many angels can fit on the head of a pin turns out to be
unanswerable when we find out there aren't any angels, or that angels
aren't the sort of thing that you can stack on a pinhead.

Now, when this happens, it doesn't mean the question wasn't worth asking
in the first place. Often the only way you can learn that the
presuppositions of your question were wrong is by asking the question
and doing your best to answer it! Gradually you realize you have hit a
dead end... and then you turn around, back out, and go somewhere new and
interesting.

One must always be open to the possibility that a given question will turn
out to have no answer... or at least, no answer *as far as we can tell*.
One shouldn't pigheadedly demand an answer after someone gently informs
one that the question has no answer. Instead, one should try ones best
to understand how the question could have no answer... and to see if it
does or not.

One learns amazing things when one learns that an interesting question has no
answer. It's an experience to be relished!

BEN

unread,
Jan 12, 1996, 3:00:00 AM1/12/96
to
In article <matthew-1201...@mattsmac.mcb.net>, mat...@mcb.net
(Matthew Jones) writes:

> If all the galaxies have flying apart ever since the big bang then
> shouldn't it be possible to work out the direction they are travelling in
> and therefore the direction that they came from? If this was done for all
> the galaxies then shouldn't these lines all cross at some point which
> would make that the center of the universe?

Indeed, you can do that, and you will find that they are all moving
directly away from us.

Any observer anywhere else will see the same thing.

So, can you now answer the question?

Ben


Bob Silverman

unread,
Jan 12, 1996, 3:00:00 AM1/12/96
to
In article <matthew-1201...@mattsmac.mcb.net>,

Matthew Jones <mat...@mcb.net> wrote:
>If all the galaxies have flying apart ever since the big bang then
>shouldn't it be possible to work out the direction they are travelling in
>and therefore the direction that they came from? If this was done for all
>the galaxies then shouldn't these lines all cross at some point which
>would make that the center of the universe?
>
>Notice that I have posed this as a question as I'm sure there is a reason
>why this is wrong but I am just curious as to why this wouldn't work.

Put some black dots on a deflated balloon. Now blow up the balloon.
The back dots all move away from each other, don't they?

The "universe" is represented by the surface of the balloon. Where
is the center? The center of the balloon is well-defined, but that
center is not part of the universe. The universe is given by only
the surface of the balloon.

--
Bob Silverman
The MathWorks Inc.
24 Prime Park Way
Natick, MA

Matthew Jones

unread,
Jan 12, 1996, 3:00:00 AM1/12/96
to
If all the galaxies have flying apart ever since the big bang then
shouldn't it be possible to work out the direction they are travelling in
and therefore the direction that they came from? If this was done for all
the galaxies then shouldn't these lines all cross at some point which
would make that the center of the universe?

Notice that I have posed this as a question as I'm sure there is a reason
why this is wrong but I am just curious as to why this wouldn't work.

Thanks in advance.

/-------------------------+------------+--------------\
| Matthew Jones | Dex on | The Truth Is |
| mat...@mcb.net | IRC | Out There |
\-------------------------+------------+--------------/

Oz

unread,
Jan 12, 1996, 3:00:00 AM1/12/96
to
ba...@guitar.ucr.edu (john baez) wrote:

>In article <30f4215f...@news.demon.co.uk> O...@upthorpe.demon.co.uk writes:
>
>>So my question. Very simple and basic. When one says space
>>is expanding, do we really mean nothing more than galaxies
>>have motion away from each other due to essentially initial
>>velocities originating at the BB, or something more, well,
>>interesting.
>

>It's rather frustrating. Ted Bunn and I explained to you pretty
>carefully that one cannot go around talking about distant galaxies
>"moving away from each other" in curved spacetime, without being fatally
>ambiguous. You seemed to get the point. Now you are going around doing
>just that!

Yes, you did. The problem is that whilst you illustrated
quite clearly why this was ambiguous, or possibly more
correctly meaningless and inappropriate, you haven't
provided me with replacement descriptive tools to better
understand or describe the concept. Without these it's
difficult to proceed further. Remember that you and Ted only
really described the Milne model, which is essentially SR in
concept so I still have to get further to get a glimmering
of how the mechanism should be viewed. Obviously it's a GR
explanation, but I have been spectacularly unsuccessful in
getting anyone to illustrate the salient differences and
ideas in a tolerably low mathematical way at least.

>The only way I could answer your "simple and basic" question would be to
>break you of the habit of talking about the relative motion of distant
>objects in curved spacetime. Breaking you of this habit, and getting
>you to understand curved spacetime, would leave you understanding how
>funny your question sounds.

Couldn't agree more. I know it's not right. What other
descriptions should I use, however?

> You would see what it means to say "space
>is expanding". You would also see why this is a slightly sill way to
>put it. You would understand the redshift and realize the pointlessness
>of trying to separate it into a "gravitational part" and a "part due to
>the relative motion of the galaxies". You would be happy and wise.

Agreed, it's why I asked. It's clearly odd that people here
say "oh the redshift can be viewed as a gravitational
redshift" and again "the redshift can be viewed as a
recessional velocity redshift". Clearly and frustratingly
neither of these is a very satisfactory explanation since
it's either one, the other, both or more likely neither
really. So I know it's a most unsatisfactory explanation.
That's why I keep on asking. Is it really and absolutely
required to do a six year postgrad course in GR to see in
overview how it comes about? Forty years ago this was
required for SR, now (and 30 years ago) it's done quite
adequately in ten one hour lectures, maths included.

Well, I rather doubt that this alone would make me happy and
wise. Happier and wiser might be a better description.

>However, maybe I am not the right teacher for you. Perhaps there is
>some way to understand the big bang while going around talking about the
>relative velocity of distant objects in curved spacetime. All I know is
>that I can't explain it to anybody before they stop talking that way.

To start with it would help to know what are an appropriate
descriptors for GR. Obviously distance and velocity are
inappropriate. OK lets abandon them for ....... ?

>So I won't try.

I sincerely hope you will reconsider this.

Paul Budnik

unread,
Jan 12, 1996, 3:00:00 AM1/12/96
to
john baez (ba...@guitar.ucr.edu) wrote:
: The unknown becomes known quite often, the unknowable less so. But

: when it does, it wasn't really the unknowable after all, so you should
: have been more careful about what you say about it.

For example you should be careful about claiming that probabilities
are irreducible. Someday someone may reduce them.
--
Paul Budnik
pa...@mtnmath.com, http://www.mtnmath.com

Edward Green

unread,
Jan 12, 1996, 3:00:00 AM1/12/96
to
'ba...@guitar.ucr.edu (john baez)' wrote:

>More precisely, we should always think of
>
>sqrt(-1) t/hbar,
>
>where t is time and hbar is Planck's constant, as just another word for
>
>1/kT,
>
>where T is temperature and k is Boltzmann's constant. The point is that
>time evolution is given (in quantum mechanics) by
>
>exp(-itH/hbar)
>
>where H is the Hamiltonian or energy operator, while the equilibrium
>state in (quantum) statistical mechanics is given by
>
>exp(-H/kT).


One of the prices for thinking of the universe as some huge logical
structure sort of unwinding in time, is that I no longer hear that spooky
holywood string section when people use concepts like "imaginary time".
You've lost that mystical feeling, whoa-oh and etc.

I *know* you didn't intend it that way; I just wonder what universe of
discourse makes these objects synonyms. You said: "it means, for example,
that quantum field theory and quantum statistical mechanics are
mathematically isomorphic." Okey doke, that is very concrete.

Here is what I think is going on.

There is a "deep" relation between energy and time. Indications of this
include their appearance together in an uncertainty relation, the
identification of frequency with energy in many cases, [symmetry under
time translation implying conservation of energy?...] You can probably
add many more yourself.

If these all sound the same, it's because at the root they are. These
clues suggest to me that the primitive of energy is the corrugation in time
of... whatever is corrugated by the wave function. We reify energy, to
borrow Jim Carr's phrase, but the "conservation of energy" really
expresses the invariance of some functional of this corrugation.

The invariance is only in the mean, so at fine enough scales we see energy
being "borrowed", and so forth, which we interpret in light of
uncertainty relations.

Alb^H^H^H Ed has spoken. ;-)

--

Ed Green
egr...@nyc.pipeline.com

Jason Kodish

unread,
Jan 12, 1996, 3:00:00 AM1/12/96
to
In article <4d1mfc$i...@jobes.sierra.net> rba...@foothill.net writes:
>
>located. When I stand outside at night and look at the Southern Cross,
>or Ursa Major, or Polaris or whatever. I want to point to one of them
>and say, "So the Center of the Universe is that way." Can't one of you
>answer that question?


It is located everywhere. To clarify that, you've got to think of all the
worldlines of all the objects in the Universe. When you do that, it becomes
in fact, clear that the correct question is not "where is the center of
the universe?" but "when is the center of the universe" and that answer
is roughly 20 billion years ago.
As for a spacial location, asking that is the same as asking where the
center of the surface of the Earth is.


>
>
>
>

--
Jason Kodish
Thirring Institute for Applied Gravitational Research
Thirring homepage: http://www.geopages.com/siliconvalley/1659
______________________________________________________________________
How can semi-intelligent people endowed with a
whole Continent of resource riches get it so
messed up. -Martin Burger


allan padgham

unread,
Jan 12, 1996, 3:00:00 AM1/12/96
to
maybe you can't get to the center of the universe
from where you are.


Sebastien Lepine

unread,
Jan 12, 1996, 3:00:00 AM1/12/96
to
mat...@mcb.net (Matthew Jones) wrote:
>If all the galaxies have flying apart ever since the big bang then
>shouldn't it be possible to work out the direction they are travelling in
>and therefore the direction that they came from? If this was done for all
>the galaxies then shouldn't these lines all cross at some point which
>would make that the center of the universe?

Astronomers observe all distant galaxies to be redshifted, wherever they are
looking in the sky. If you extrapolate their "movement" backwards, you reach
the conclusion that YOU ARE the center of the universe, because all their
"travelling lines" cross just where we are (apart from some small deviations
that are due to the local movement of our galaxy); every distant galaxy is
exactly going away from us.

However, you should reach the same conclusion wherever you are in the universe!

Why? Because the galaxies are not travelling away from some definite point
(like shrapnels from an exploding grenade), but rather going AWAY
FROM EACH OTHER (like raisins in a rising cake).

That's why the question "Where is the center of the universe?" is so subtle,
because the "center" is (if we refer to the recession of galaxies) everywhere
at the same time.

--
Sebastien Lepine


Hannu Poropudas

unread,
Jan 12, 1996, 3:00:00 AM1/12/96
to
In article which is dated 11 jan 1996 16:59:52 GMT
v...@clinet.fi (Ville Sinkko) wrote:

>Just listen for the sound of the flutes...;)


Anisotropy of cosmic background radiation could also
indicate very slow rotational angular frequency of
the Universe.?

How this kind rotation is possible if it does
not happen as rotation about the center of space.?

The topology of the Universe may be weird and complicated,
it is possible also question about 10 dimensional curved
space, where 6 dimensions could be curled up into very
small scale and we know nothing about them (and possible
we will never know much about them as I have understood),
so it could be safe always not to say anything certain
about structure of the Universe.?

I would also to remind that I'am only an amateur physicist
and that is why my understanding about these matters is far
from complete and my writings could contain my own errors.

--


Best Regards,

Hannu Poropudas.

"It's Not What You Know That Matters
... It's Knowing What You Don't."


Emory F. Bunn

unread,
Jan 12, 1996, 3:00:00 AM1/12/96
to
In article <4d3vbv$5...@henge2.henge.com>,

Tom Gossman <gos...@hcob.wmich.edu> wrote:
>An assertion that "it is currently believed (by most cosmologists)" and
>another assertion that the question is just plain nonsense make me very
>suspicious.

I would tend to avoid saying that the question of where the center of


the Universe is is "nonsense," since that's such a pejorative term,
and you have to have studied the subject some before you understand
why it's not a good question to ask. It turns out, however, that it's
not a good question to ask. Specifically, it appears to be a question
without an answer.

>It seems logical that when we had the Big Bang, and space was

>created forming out in all directions, it all began at some point which is
>still there in that created space.

I don't know what you mean by "logical" here. That certainly is a
logical possibility; it's also a logical possibility that there is no
such point. I think what we're talking about here is not logical
possibility but intuitive plausibility. To many people the notion of
expansion without a center is intuitively hard to grasp or
implausible. That's perfectly natural, but it's important to remember
that the Universe is not required to behave in ways that please our
intuition.

>Of course science can be contrary to
>common logic, but so far I have seen noting to indicate that there is any
>accpted theory (beyond assertions) that such is the case here.

Again, I think you mean something more like "intuition" than "logic"
here. I'm unaware of any scientific theories that are intrinsically
illogical, but many are counterintuitive to the uninitiated. Quantum
mechanics and relativity are the classic examples; the big bang
seems to be another.

>As to the questions being nonsense, I disagree. It is a perfectly rational
>question, unlike a question about the color of time, etc.

I agree. It's a rational question that happens to have no answer, as
far as we know.

-Ted

john baez

unread,
Jan 12, 1996, 3:00:00 AM1/12/96
to

>So my question. Very simple and basic. When one says space
>is expanding, do we really mean nothing more than galaxies
>have motion away from each other due to essentially initial
>velocities originating at the BB, or something more, well,
>interesting.

It's rather frustrating. Ted Bunn and I explained to you pretty
carefully that one cannot go around talking about distant galaxies
"moving away from each other" in curved spacetime, without being fatally
ambiguous. You seemed to get the point. Now you are going around doing
just that!

The only way I could answer your "simple and basic" question would be to


break you of the habit of talking about the relative motion of distant
objects in curved spacetime. Breaking you of this habit, and getting
you to understand curved spacetime, would leave you understanding how

funny your question sounds. You would see what it means to say "space


is expanding". You would also see why this is a slightly sill way to
put it. You would understand the redshift and realize the pointlessness
of trying to separate it into a "gravitational part" and a "part due to
the relative motion of the galaxies". You would be happy and wise.

However, maybe I am not the right teacher for you. Perhaps there is


some way to understand the big bang while going around talking about the
relative velocity of distant objects in curved spacetime. All I know is
that I can't explain it to anybody before they stop talking that way.

Vertner Vergon

unread,
Jan 13, 1996, 3:00:00 AM1/13/96
to
john baez (ba...@guitar.ucr.edu) wrote:
: In article <4ctt2p$g...@pipe10.nyc.pipeline.com> egr...@nyc.pipeline.com (Edward Green) writes:
:
: >Another symmetry principle? The EM field is invariant under simultanteous
: >scaling of space and shift in the energy scale?
:
: >This still leaves me with the nagging question, is *this* a cosmic
: >coincidence, or a key to something deeper.
:
: >This time I don't really expect an answer.
:
: Come, come: you expect far too little. As I noted before, rescaling
: time is the same as rescaling temperature, quite generally. What's
: special about the electromagnetic field is that it's "conformally
: invariant". This implies that if you have any solution of Maxwell's
: equations, and you simultaneously rescale space and time by the same
: factor, you get a new solution.
:
: It turns out that all the field equations of massless particles are
: conformally invariant, but not the field equations of any massive
: particles.

Rather than "massless particles" could not one consider "mass in a
different state"?

I see now why you chaps cling so desperately to massless particles. -- But
you have to find a different way because massless particles 'aint' :-)
not any more than 'timeless seconds' or 'anhydrous water'.

*Everything* in the universe consists of matter (mass). There is either
matter or space between matter. (That's the simple beauty of the universe.)
All else is anthropocentric. If you do not have matter, you have nothing.

Of course there are those that answer, that is an "old fashioned outmoded
view", etc. But the problem is they have nothing to offer in its place
except mathematical represetations in which the derivations coincide with
empiricism but that does not necessarily mean that the equation
*representations* are accurate. (I wonder if I am saying this clearly.)

On these posts, I have asked many to describe *physically* a massless
particle -- no answer.I have asked them to describe the physical
composition of massless energy -- no answer. I've asked them to explain
how a massless particle produces mass merely by being 'stopped' (speaking
of pair production) -- no answer.

I have shown that one can attribute mass to the photon and show that
there is a distinct difference in this mass and the mass of a particle
(which I and Einstein refer to as "ponderable: mass)

Ex. E = h nu = m_phot c^2 which is of the form mc^2 whereas for a

massive particle E = mc^2(gamma -1).

Likewise, I suspect that (if you really wanted to) you could accomodate
your interpretations of field equations to consider a photonic mass, but
of different characteristics than ponderable particles, i.e., massive
particles. (But I don't think that's likely to happen.) It's easier to
call Vergon a nut. :-) Oh well, such is life.
- - - - - - - -

I read your piece below on the Compton wavelength --- *Excellent*, top
quality. You are an excellent expositor.

Regards,

VERTNER VERGON

Vertner Vergon

unread,
Jan 13, 1996, 3:00:00 AM1/13/96
to
john baez (ba...@guitar.ucr.edu) wrote:
: In article <4cp7jr$f...@pipe11.nyc.pipeline.com> egr...@nyc.pipeline.com (Edward Green) writes:
:
: >I'd still like to know if the red-shift/temperature-shift equivalence in
: >black-body spectrums is some incredible cosmic coincidence, or the source
: >of deep insight. Does anybody have an opinion on that?

<Deletion>
:
: A similar relationship holds in classical mechanics, but as usual
: classical mechanics is more complicated than quantum mechanics, so
: I will only discuss the quantum case. (The real world is quantum, too,
: so if we are going to talk about just one case let's talk about the real
: one.)
:
: This relationship is the basis of all sorts of wonderful things; it


: means, for example, that quantum field theory and quantum statistical

: mechanics are mathematically isomorphic. This is a very practical
: isomorphism that physicists use all the time. It's also the basis of
: Hawking's work on "imaginary time" and the statistical mechanics of
: black holes.
:
: In the present case, the point is that redshifts are due to *time
: dilation*: the atoms wiggling in the receding star appear to be moving
: slower to us. By the above, *time dilation* is equivalent to
: "temperature dilation": the light from the receding star appears cooler
: to us.

What about doppler shifts? How does that fit in?

V.V.

Vertner Vergon

unread,
Jan 13, 1996, 3:00:00 AM1/13/96
to
Bruce Scott TOK (b...@ipp-garching.mpg.de) wrote:
: Edward Green (egr...@nyc.pipeline.com) wrote:
:
: [...]
:
: : The distribution of modes of the EM field is self-similar under shifts of
: : the energy scale.
:
: More or less what I said.
:
: [...]
:
: : This still leaves me with the nagging question, is *this* a cosmic
: : coincidence, or a key to something deeper.
:
: Just basic thermodynamics and the way temperature is defined. If you
: want, the self-similarity is a consequence of the fact that the energy
: of a photon is inversely proportional to wavelength, or...
:
: its energy is its momentum times c, or...
:
: photons are massless!
:
:
: Let's see if Vergon jumps in :-)

Yes, I'm watching.
V.V. ;-)

:
: --
: Mach's gut!
: Bruce Scott The deadliest bullshit is
: Max-Planck-Institut fuer Plasmaphysik odorless and transparent
: b...@ipp-garching.mpg.de -- W Gibson

Vertner Vergon

unread,
Jan 13, 1996, 3:00:00 AM1/13/96
to
john baez (ba...@guitar.ucr.edu) wrote:
: In article <4cp7jr$f...@pipe11.nyc.pipeline.com> egr...@nyc.pipeline.com (Edward Green) writes:
:
: >I'd still like to know if the red-shift/temperature-shift equivalence in
: >black-body spectrums is some incredible cosmic coincidence, or the source
: >of deep insight. Does anybody have an opinion on that?
:
: We should immediately assume that it's not a coincidence, since there
: are very few coincidences of this sort. The question is then simply
: whether it's a "deep" fact or a "shallow" one.
:
: Actually, while not especially deep itself, it's a spinoff of the deep
: fact that imaginary time and inverse temperature are two ways of talking
: about the same thing!
:
: More precisely, we should always think of
:
: sqrt(-1) t/hbar,
:
: where t is time and hbar is Planck's constant, as just another word for
:
: 1/kT,

Now we have "inverse time" -- I presume your unit of time is the second.
What is so sacred about the second? If the second were of a different
length then your whole show is out of whack -- kaput.

That is true for h also as one of the dimensions is time.

Can you explain that?


: where T is temperature and k is Boltzmann's constant. The point is that


: time evolution is given (in quantum mechanics) by

^^^^^
WHAT time?

: exp(-itH/hbar)


:
: where H is the Hamiltonian or energy operator, while the equilibrium

Energy is expressed by the erg -- which entails time as a function.
What if the second was a different length of time?
Your whole structure depends on this and is therefore invalid. ;-)

(That's what you people have been saying about my quantum size.
So what's sauce for the goose ...)

V.V.

keith stein

unread,
Jan 13, 1996, 3:00:00 AM1/13/96
to
i just wondered if these quotes from Prof Boeyens could be germane ?

'It is therefore wrong to think that in some areas it [science]
has reached such abstract levels that it is no longer freely accessible
to the layman. This only happens where CONFUSION REIGNS and where it
becomes benificial to HIDE A LACK OF UNDERSTANDING UNDER A CLOUD of
OBFUSCATION or tedious mathematics. Useful ideas in science are all
SIMPLE AND REDUCIBLE TO PEDESTRIAN DISCOURSE.'
Professor Jan Boeyens
Dean of the Faculty of Science,
Witwatersrand University,
Johannesburg.


And the good professor continues......

'By its very nature science has no final answers, NO AUTHORITIES,
and no enduring theories. It thrives by constantly challenging all
postulates, conjectures, theories and hypotheses.........'

'An unfortunate new complicating factor emerging into contempary
physics is the ARROGANT NOTION that FINAL ANSWERS CAN AT LAST BE
PROVIDED through consistent application of so-called standard models.
There is a standard model for cosmology and a standard model for
elementay particle theory - each one supporting the other. The symbiosis,
however, is so cosy that NO DISSENTING VIEWS CAN BE TOLERATED.......
The dogmatic appeal to the standard big-bang model has become so powerful
that experimental observation is becoming a nuisance. Conflicting
new data are simply neutralised by adding another untestable......'


EMPHASIS ONLY added by,
keith stein

Wilbert Dijkhof

unread,
Jan 13, 1996, 3:00:00 AM1/13/96
to
Jeff....@jet.uk (Jeff Candy) wrote:

>gos...@hcob.wmich.edu (Tom Gossman) wrote:
>>In article <4d24ih$7...@agate.berkeley.edu>,
>> t...@physics12.Berkeley.EDU (Emory F. Bunn) wrote:
>>>
>>>Or maybe you aren't listening to the answers. The answer, as several
>>>of us have said, is that as far as we know there is no such place.
>>>
>>In other words, it isn't that there is no such place. Its just that they
>>can't answer your question. The original question is a fair one, however.
>
>Before you can ask, "what is the centre of the universe?",
>you have to define the "centre of the universe". The geometry
>of space-time is more complicated than the flat 3-D space which
>forms the basis of most people's intuition, and an analogue to
>the concept "centre" not immediately obvious. Define the concept
>of centre and we'll tell you the answer, or point out how it can be
>estimated. By one definition, the "centre" is at t=0+, for reasons
>which have already been discussed.
>
>This thread reminded me of the impossible-to-explain (sometimes)
>question "what happened before the big bang?". Since the space-time
>manifold is concieved to be a set open at t=0 (i.e., the universe
>exists only for t>0), the question is ill-posed. This answer
>typically illicits the response, "yeah, but what happened before
>that???".
>
>
>
>Jeff Candy ... man -- every man -- is an end
>Analytic Theory Group in himself, not the means to the
>JET Joint Undertaking ends of others ...
> --- Ayn Rand
>
>
>===============================================================================
> The above article is the personal view of the poster and should not be
> considered as an official comment from the JET Joint Undertaking
>===============================================================================

Ever heard of:

- Superstringtheory : The Big-Bang (didn't start at t=0 but at t=10^(-43)) is
by-product of the collapsing of the 10 dimensional universe
in a 4 and 6 dimensional one.
- The oscillating universe: There happened indeed something (= Big-Crunch of the former
universe) at/before t=0.

* The 4-dimesional universe didn't exist only for t>0, but since 10^(-43) since
time is quantizited. The 10-dimensional universe hovever did exist between t=0 and
t=10^(-43).

WDijkhof


gl...@gnn.com

unread,
Jan 13, 1996, 3:00:00 AM1/13/96
to

In article <4d99ru$q...@marina.cinenet.net> Vertner Vergon wrote:
>*However*, without Al leading the way with his great insight and
>use of math, I never could have or would have done my bit.
>
Since Al never accurately derived the Poincare' equations
(Lorentz Transformations Equations); and since Al made so many
mathematical errors in his 1905 paper that they cumulately wipe out
these equations, it is clear that VV's "bit" is just as defective
as the false leader he followed.


Glird http://members.gnn.com/glird/reality.htm


Edward Green

unread,
Jan 13, 1996, 3:00:00 AM1/13/96
to
'schm...@fermi.wias-berlin.de (Ilja Schmelzer)' wrote:

>
>But the metric of the sphere has a symmetry - of rotations. The plane
>has also a symmetry. They are different. Thus, the symmetry group of
>the image of this homeomorphism will be smaller - the intersection
>between the two symmetry groups. There will be no translational
>symmetry for this solution. There is one special point - the South
>pole, which may be considered as the center of this solution. No
>boundary.

So you assert that PG does *not* require translational invariance of space?

Very stange to me. I assume you picked this particular homeomorphism
because it is easy to visualize. I also assume there is a similar
construction in 4-space mapping a closed three dimensional universe onto
3-space.

>How to distinguish these two interpretations by measurement I don't
>know. We cannot prove by experiment that all points are really
>present.

Yes. But can we in principle find by experiment whether the portion of
space near us in homogeneous, and in particular, preserves physical laws
under translations?

--

Ed Green
egr...@nyc.pipeline.com

Edward Green

unread,
Jan 13, 1996, 3:00:00 AM1/13/96
to
'gos...@hcob.wmich.edu (Tom Gossman)' wrote:

>>Or maybe you aren't listening to the answers. The answer, as several
>>of us have said, is that as far as we know there is no such place.
>>
>In other words, it isn't that there is no such place. Its just that they

>can't answer your question. The original question is a fair one, however.


"As far as we know there is no such place". What's wrong with that? This
isn't "stump the physicists". There are lots of questions you could ask
that would have a similar answer.

A more detailed answer would have listed the assumptions necessary to have
a "center" in some sense, and asserted that most currently accepted models
don't meet these assumptions, but it is not absolutely required by
observation that the universe may not in fact meet them.

In fact, if you sift through the whole thread, I think that is pretty
much the answer that was given. If you don't like it, there is no need to
go away sulking "Aw, gee. They couldn't answer the question!".

--

Ed Green
egr...@nyc.pipeline.com

It is loading more messages.
0 new messages